UKITE 2011

156
Question 1 Which of the following drugs is a direct factor Xa inhibitor? Phenindione Rivaroxaban Aspirin Clopidogrel Dalteparin References http://en.wikipedia.org/wiki/Anticoagulant A Once-Daily, Oral, Direct Factor Xa Inhibitor, Rivaroxaban (BAY 59-7939), for Thromboprophylaxis After Total Hip Replacement Bengt I. Eriksson et al Circulation. 2006;114:2374- 2381 Question 2 The following statement regarding 1984 modification of Gustilo and Anderson's classification of open fractures is true: Type II injuries are heavily contaminated Type IIIB injuries have adequate soft tissue cover Type I injuries have a wound less than 2 cm long Type IIIA injuries have adequate soft tissue cover despite extensive lacerations All arterial injuries are graded Type IIIC regardless whether the arterial injury requires repair or not.

description

FRCS Orth

Transcript of UKITE 2011

Page 1: UKITE 2011

Question 1

Which of the following drugs is a direct factor Xa inhibitor?

Phenindione

Rivaroxaban

Aspirin

Clopidogrel

Dalteparin

Referenceshttp://en.wikipedia.org/wiki/Anticoagulant

A Once-Daily, Oral, Direct Factor Xa Inhibitor, Rivaroxaban (BAY 59-7939), for Thromboprophylaxis After Total Hip Replacement Bengt I. Eriksson et al Circulation. 2006;114:2374-2381

Question 2

The following statement regarding 1984 modification of Gustilo and Anderson's classification of open fractures is true:

Type II injuries are heavily contaminated

Type IIIB injuries have adequate soft tissue cover

Type I injuries have a wound less than 2 cm longType IIIA injuries have adequate soft tissue cover despite extensive

lacerationsAll arterial injuries are graded Type IIIC regardless whether the arterial

injury requires repair or not.

ReferencesType III was reclassified into A,B and C in the 1984 modification. Gustilo, RB., Mendoza RM., Williams DN. Problems in the management of type III open fractures: a new classification. journal of trauma 24:742-746, 1984

J Orthop Trauma. 1993;7(4):357-60.Interobserver reliability in the Gustilo and Anderson classification of open fractures.Horn BD, Rettig ME

Page 2: UKITE 2011

J Am Acad Orthop Surg. 2010 Jan;18(1):10-9.Open tibial shaft fractures: I. Evaluation and initial wound management.Melvin JS, Dombroski DG, Torbert JT, Kovach SJ, Esterhai JL, Mehta S.

Open fracture classification after Gustilo and Anderson

Grade ISmall in-to-out wound (< 1 cm). Low-energy injury, no soft-tissue crushingor stripping, no bony comminution.

Grade II

Laceration over fracture (1-10 cm wound). Low-energy injury, minimal soft-tissuecrushing or stripping, no bony comminution.

Grade IIIAGrade IIIBGrade IIIC

High-energy injury, bony comminution or segmental fracture, soft-tissue crushingstripping or serious contamination but adequate soft-tissue cover after debridement.High-energy injury, bony comminution or segmental fracture, soft-tissue crushingstripping or serious contamination, inadequate soft-tissue cover after d6bridementAny open fracture associated with a local vascular injury requiring repair.the severity of the wound to other clinicians and thusprevent repeated and potentially contaminatingwound examinations outside of a sterile environment.

SURGICAL CLASSIFICATIONFormal classification of the open fracture occurs aftersurgical debridement. The most widely used system isthat attributed to Gustilo and Anderson. 7,8 This classifiesthe injury into three types, with the high-energyinjuries subdivided into three further groups (Table 2).This system has stood the test of time and is widely

Page 3: UKITE 2011

understood. The essential elements being the differentiationbetween high- and low-energy injuries and theconcept of adequacy of soft-tissue cover in the highenergyinjuries.The site of the fracture has an important effect onthe injury grade, particularly with regard to quantityof local muscle cover and the vulnerability of the localsoft tissue to injury. A fracture in the normal femurhas to be high-energy because of the intrinsic strengthof the bone; accordingly all open femur fracturesmust be grade III, although the thick muscle cover ofthe thigh usually provides adequate soft-tissue cover.In comparison, the tibia is unique with regard to itspoor soft-tissue cover and particular vulnerability toinjury. Accordingly, it is difficult to have a grade IIIainjury to the tibial diaphysis where most high-energydiaphyseal and distal tibial fractures are grade IIIband require complex reconstructive surgery. The mostsevere injury (IIIc) is the open fracture associated witha vascular injury that requires repair. These can occurwith either a severe or very a relatively 'minor' would.The high grading is because of the primary risk to thelimb associated with the arterial disruption and thesusceptibility of ischaemic muscle to infection.A similar classification system for closed fractureshas been proposed by Tscherne. 4 This also describesthe range of soft-tissue injuries from essentially nosignificant injury to major vascular disruption, severemuscle revitalization or compartment syndrome, allwithin a closed skin envelope, a fuller classificationsystem considering both injury types is provided bythe comprehensive classification from the AO. 9 Thisindividually classifies the integument, nerve and vasculartissues producing a more comprehensive anddetailed system

Page 4: UKITE 2011

Question 3

Of the following conditions, which is caused by a point mutation for the gene encoding fibroblast growth factor receptor 3 (FGFR3)?

Ehlers Danlos Syndrome

Marfan's Syndrome

Duchenne's Muscular Dystrophy

Achondroplasia

Becker Muscular Dystrophy

Referenceswww.orthoteers.comwww.wheelessonline.comAutosomal dominant disorder.Arises due to a point mutation for the gene encoding a fibroblast growth factor (Fibroblast growth factor receptor 3).over 80% of all persons with this disorder are born to parents who are not achondroplastic, & such parents rarely have second achondroplastic child.It is presumed in these instances that disorder is consequence of mutation and may be related to a mutation in a fibroblast growth factor gene.

Question 4

Which of the following is NOT a common risk factor for the development of hypertrophic and keloid scars?

Deep skin wounds

Dark pigmented skin

Old age

Wound tension

Delayed healing

ReferencesEssential revision notes for the intercollegiate MRCS

C. Parchment SmithAetiology and management of hypertrophic scars and keloids.O'Sullivan ST, O'Shaughnessy M, O'Connor TP.

Page 5: UKITE 2011

Ann R Coll Surg Engl. 1996 May;78(3 ( Pt 1)):168-75.

Hypertrophic scars and keloids can be described as variations of typical wound healing. In a typical wound, anabolic and catabolic processes achieve equilibrium approximately 6-8 weeks after the original injury. At this stage, the strength of the wound is approximately 30-40% that of healthy skin. As the scar matures, the tensile strength of the scar improves as a result of progressive cross-linking of collagen fibers. At this point, the scar is usually hyperemic and it may be thickened, but it tends to subside gradually over months until a flat, white, pliable, possibly stretched, mature scar has developed. When an imbalance occurs between the anabolic and catabolic phases of the healing process, more collagen is produced than is degraded, and the scar grows in all directions. The scar is elevated above the skin and remains hyperemic. Excessive fibrous tissue is classified as either a keloid or a hypertrophic scar.

Question 5

Which of the following describes the relationship between intensity of an x-ray beam and the distance from its source?

Attenuation coefficient

Bragg's law

Compton effect

Inverse square law

Photoelectric effect

ReferencesManoj Ramachandran, The Stanmore guide: Pg 51The intesity of an x-ray is indirectly related to the square of the distance from its source - the inverse square law

Question 6

Regarding obtaining consent for an operative intervention, which of the following statements is true? The refusal of a competent person aged 16-17 can never be over-ridden by

either a person with parental responsibility or a court.

Page 6: UKITE 2011

Refusal by a competent child and all persons with parental responsibility for the child can be over-ruled by the court if the welfare of the child so

requires. A parent or relative can give consent on behalf of an adult unable to give

consent for him or herself (an “incapable” adult). When a patient gives valid consent to an intervention, in general that consent remains valid for a period of four weeks after which time it must be re-

obtained. Consent may only be obtained by the clinician providing the treatment or

investigation

ReferencesReference guide to consent for examination or treatment, March 2001http://www.gosh.nhs.uk/clinical_information/clinical_guidelines/cpg_guideline_00151

Question 7

According to the Musculoskeletal Tumour Society, excision of a tumour through the pseudo-capsule of a tumour (the reactive zone) is described as which of the following?

Amputation

Intra-lesional

Marginal

Radical

Wide

ReferencesSim FH, Frassica DA: Soft tissue tumours: Diagnosis, evaluation and management. AAOS 2:209, 1994Surgical margins are graded according to MSTS as follows:Intralesional: Plane of dissection is through tumour. Marginal margin: Marginal line of resection through reactive zone of tumour. Wide margin: Entire tumour is removed with cuff of normal tissue. Radical margin: Entire tumour and compartment is removed

Question 8

Page 7: UKITE 2011

Which of the following accounts for 65-80% of the dry mass of flexor tendons?

Collagen type I

Collagen type II

Collagen type III

Collagen type IV

Elastin

ReferencesCollagen One: skin, tendon, vascular, ligature, organs, bone (main component of bone)Collagen Two: cartilage (main component of cartilage)Collagen Three: reticulate (main component of reticular fibers)Collagen Four: forms bases of cell basement membrane

Current Concepts Review Tendon Injury and Tendinopathy: Healing and Repair Pankaj Sharma, Nicola MaffulliThe Journal of Bone and Joint Surgery (American). 2005;87:187-202

Question 9

Mayfield stage four peri-lunate injury is characterised by which of the following

Luno-Triquetral injury

Scapho-Lunate injury

Lunate dislocation

Luno-Capitate injury

Piso-Triquetral injury

ReferencesGreen's Operative Hand Surgery 5th Edition, Page 542-543. Mayfield's Classisfication describes a consecutive pattern of peri-lunate joint disrutption. Stage I: Scapho-LunateStage II: Luno-Capitate

Page 8: UKITE 2011

Stage III: Luno-triquetralStage IV: Lunate dislocation

Question 10

With reference to spica cast treatment of femoral shaft fractures in a 5 year old child

25 degrees of coronal plane deformity is acceptable.

Femoral shortening of 4 cm is acceptable due to later over-growth.

25 degrees of sagittal plane deformity is acceptable.The more proximal the fracture, the lesser the hip flexion required in the

spica.

20 degrees of rotational deformity is acceptable.

ReferencesAcceptable alignment depends upon the child's age but in general, no more than 15 degrees of deformity in the coronal plane and 25 to 30 degrees in the sagittal plane is acceptable. Shortening should not exceed 2 cm.The hips should be flexed to 60 to 90 degrees to align the distal fragment with the flexed proximal fragment. The higher the fracture, the more the hips should be flexed Spica cast treatment is appropriate only for children upto the age of 10 years. For a 11 year old patient, internal (plates, rods) or external fixation options should be considered

Tachdjian's Pediatric Orthopaedics. 4th edition. 2008. Lower Extremity Injuries. Femur. pp 263

Question 11

Page 9: UKITE 2011

Clotting studies on a patient with haemophilia A are most likely to show the following

Prolonged APTT and PT

Prolonged APTT and normal PT

Prolonged APTT and reduced PT

Normal APTT and prolonged PT

Reduced APTT and prolonged PTReferencesProthrombin time is measure of extrinsic pathway of coagulation and activated partial thromboplastin time is measure of intrinsic pathway of coagulation. Heamophila A accounts for 80% of all heamophilias and is due to factor VIII deficiency.Prolonged APTT and PT: Factor V,X deficiency, DIC, WarfarinProlonged APTT and normal PT: Hemophilia, Von Willebrand's diseaseNormal APTT and prolonged PT: early liver failure www.gpnotebook.comwww.wheelesonline.com

Question 12

In Class II hypovolaemic shock the characteristic finding would be

Systolic BP falls to 100mm Hg or less

Tachycardia > 100/min

30-40% blood loss

Urine output < 15mls/hour

Pulse pressure is increased

ReferencesAdvanced Trauma Life Support Student Course Manual 8th EditionA class II haemorrhagic shock results in a blood loss of around 750-1500 mls (15-30% of blood volume). In this category of shock, the HR is around 100-120, BP is normal, Pulse pressure is decreased, RR is around 20-30, urine output is around 20-30 ml/hr, and the patient is mildly anxious.

Question 13

Page 10: UKITE 2011

A patient attends for a routine 3 month postoperative check following a total hip arthroplasty performed via a lateral approach. He is noted to have a positive trendelenburg sign. This is most likely due to damage to which nerve?

Inferior gluteal nerve

Superior gluteal nerve

Sciatic nerve

Nerve to piriformis

Obturator nerve

ReferencesMiller, 4th ed.

This is a simple question testing both knowledge that trendelenburg sign is caused by weak abductors and that these muscles are supplied by sup. gluteal nerve

Arises from the lumbo-sacral plexus, arises from posterior branches L4, L5, S1exits the pelvis and enters the gluteal regioin through the upper margin of the greater sciatic notch, just superior to the piriformis muscle. It courses with superior gluteal artery between gluteus medius and minimus, and supplies motor branches to both as well as the tensor fascia lata.

Two patterns of neural branching were established. The points of termination of all branches formed an arcuate pattern along the middle one-third of the deep surface of the gluteus medius muscle. The so-called safe area of the gluteus medius muscle was found to be as much as five centimeters adjacent to the greater trochanter. If this distance is not exceeded by the intramuscular incision, the risk to the superior gluteal nerve and its branches will be minimum.

Question 14

Page 11: UKITE 2011

A patient has neurogenic shock. Which of the following is the likely clinical finding?

Cessation of neurogenic shock is heralded by return of bulbocavernosus

reflex

Systemic hypotension with tachycardia following spinal cord injury

Systemic hypotension with bradycardia following spinal cord injury

Cold, moist skin

Loss of parasympathetic tone

Referenceshttp://emedicine.medscape.com/article/793582-overview (good overview article)

n.b Wheeless Textbook has confusing terminology!Neurogenic shock is a type of shock caused by the sudden loss of the autonomic nervous system signals to the smooth muscle in vessel walls. It usually occurs above the level of T6 (outflow between C7 & L1).

Trauma can cause a concussion like injury to spinal cord which leads to total sensory and motor power loss and loss of all reflexes for some period followed by gradual recovery of reflexes.This state is known as spinal shock.

It is clinically important to distinguish hypovolaemic / spinal / neurogenic shock.

1) Bulbocavernosus reflex return heralds the end of spinal shock2) Hypotension with tachycardia is hypovolaemic shock3) Loss of sympathetic tone is found in neurogenic shock4) Warm, dry skin ( no peripheral vasoconstriction)

Question 15

A femoral varus osteotomy increases which of the following

Page 12: UKITE 2011

Abductor lever arm

Adductor lever arm

More vertical line of action of the abductors

No effect

Weight bearing axis

ReferencesLovell and Winter's pediatric orthopaedics, Volume 2 By Wood W. Lovell, Robert B. Winter, Raymond T. Morrissy, Stuart L. WeinsteinPage 1072

Question 16

A 65-year old man has a suspected septic arthritis of his elbow. What are the best landmarks used for aspirating his joint?

Proximal and medial to medial epicondyle and tip of olecranon

Anterior and distal to medial epicondyle

Proximal to tip of olecranon

Between radial head, lateral epicondyle and tip of the olecranon

Anterior to lateral epicondyle

ReferencesWheeless (http://www.wheelessonline.com/ortho/aspiration_of_elbow_joint)

Discussion:    - elbow can be entered either ulnarly or radially, but radial approach is preferred in order to avoid ulnar nerve injury;    - radial nerve is lies anterior to the elbow joint, and may be moved even more anteriorly by supinating the forearm;    - landmarks for aspiration of the elbow joint are the radial head, lateral epicondyle, and tip of the olecranon (aconeus triangle);    - prior to needle insertion, elbow flexed and pronated to protect the radial

Page 13: UKITE 2011

nerve;    - 18 gauge needle is then place into the joint thru the soft spot;    - w/ this approach, needle will penetrate only the anconeus & capsule

Question 17

Which of the following does NOT lead to a thrombotic tendency?

Factor V Leiden

Antithrombin deficiency

Protein C and S deficiency

Heparin induced thrombocytopenia

Anti-Protein Z deficiency

ReferencesAnesth Analg. 2010 Nov 16. Etiology and Assessment of Hypercoagulability with Lessons from Heparin-Induced Thrombocytopenia.Sniecinski RM, Hursting MJ, Paidas MJ, Levy JH.

Thromb Res. 2008;121(6):727-34.Protein Z levels and anti-protein Z antibodies in patients with arterial and venous thrombosis.Pardos-Gea J, Ordi-Ros J, Serrano S, Balada E, Nicolau I, Vilardell M.

Question 18

Page 14: UKITE 2011

Which is the gold standard test for diagnosing post-operative Deep Vein Thrombosis (DVT)?

Duplex Ultrasonography

125 I labelled fibrinogen

D-Dimer

Venography

Impedance Plethysmography

ReferencesDuplex: good for proximal thrombi but poor for calf or pelvis, cheap and non invasive therefore most commonVenography: gold standard, used in most clinical trials but costly and invasive D Dimer: very non specific, costly and high in postoperative settingAAOS Comprehensive orthopaedic review page 155

Question 19

Which of the following suture materials is non-absorbable?

Polyglactin

Polybutester

Polyglyconate

Polydioxanone

Polyglycolic acid

ReferencesBasic plastic surgery techniques and principles: Choosing the right suture material Bayat A:student BMJ 2003;11:131-174 May ISSN 0966-6494Polybutester (Novofil) is a non absorbable suture with more compliance and elasticity than polpropylene. The other materials mentioned are biodegradable polymers.

Absorbable natural sutures -CollagenPlain surgical gutFast-absorbing surgical gutChromic surgical gut

Page 15: UKITE 2011

Absorbable synthetic sutures - Polyglactin 910 (Vicryl)Polycaprolate (Dexon II)Poliglecaprone 25 (Monocryl)PolysorbPolydioxanone (PDS II)MaxonV-Loc

Absorbable monofilament sutures -CaprosynAbsorbable multifilament sutures

Nonabsorbable natural sutures -Surgical silkSurgical cottonSurgical steel

Nonabsorbable synthetic sutures -NylonPolyester fiber (Mersilene/Surgidac [uncoated] and Ethibond/Ti-cron [coated])Polybutester Suture (Novafil)Coated Polybutester Suture (Vascufil)Polypropylene (Prolene)Surgipro II

Question 20

Page 16: UKITE 2011

The Ideberg classification is useful when planning surgical treatment of glenoid fractures. A horizontal fracture through the glenoid and scapular body with no additional glenoid fracture comprises an Ideberg type:

I

II

III

IV

V

ReferencesIdeberg R. Fractures of the scapula involving the glenoid fossa. In: Bateman JE, Welsh RP, eds. Surgery of the shoulder. Philadelphia:Decker, 1984:63-6. Ideberg Classification- Type I- Anterior avulsion fractures- Type II- Tranverse, inferior glenoid- Type III- Tranverse, superior glenoid- Type IV- Transverse, through body- Type V- Combo of types II and IV

Question 21

The typical anterior approach to the cervical spine uses the plane between which of the following structures?

The carotid artery (laterally) and jugular vein (medially)

The carotid sheath (laterally) and the trachea and esophagus (medially)

The inferior thyroid artery (medially) and the carotid sheath (laterally)

The brachial plexus (laterally) and the carotid sheath (medially)

The trachea (laterally) and the esophagus (medially)ReferencesHoppenfeld & DeBoer - Surgical Exposures in Orthopaedics: The Anatomic Approach (Publishers - LWW)

There are 3 layers of fascia which govern the anterior approach to the neck. The first is the investing layer of deep fascia (the most superficial) which

Page 17: UKITE 2011

envelopes sternocleidomastoid and surrounds the neck like a collar. Incise this to retract SCM laterally. After you go through this then in order to retract the carotid sheath laterally you need to incise the pretracheal fascia which is continous with the carotid sheath. Finally the prevertebral fascia overlies the cervical spine.

Question 22

Which one of the following statements is FALSE about trigger thumbs in childhood:

If present at birth, 30% resolve spontaneously in one year

Over the age of one year, less than 10% resolve spontaneously.

30% are bilateral.Surgical release of the A1 pulley is the treatment of choice in children over

the age of one year.

There is a strong familial inheritance pattern.

ReferencesThere is no familial inheritance pattern. Rest of the statements are correct.Trigger digits on the other hand are seen with neurologic syndromes (Trisomy 18) and mucopolysaccharidoses.Transverse incision in the digital crease is to be preferred for a better scar.

Lovell and Winter's Pediatric Orthopaedics, 6th Edn 2006. The Upper Limb, pp 922-985.

Wheeless online -- if trigger thumb is present at birth, approximately 30 % of children will recover spontaneously in one year; - 12 % of the trigger thumbs that develop at the age of six to 30 months recover in six months; - if trigger thumb develops in a child over three years of age, however, it almost never improves spontaneously; - a child not seen until after the age of four has a 50 per cent chance of developing a permanent flexion contracture

Question 23

Page 18: UKITE 2011

A patient is having a trans-articular C1/C2 screw fixation. Which structure is most at risk?

C2 nerve root

Hypoglossal nerve

Carotid artery

Vertebral artery

Vagus nerve

ReferencesJ.P.Stannard. Surgical treatment of orthopaedic trauma. 2007, pg122-123

Anterolateral C1-C2 Transarticular Fixation for Atlantoaxial Arthrodesis: Landmarks, Working Area, and Angles of Approach. Cavalcanti et alNeurosurgery 2010 Sept 67(3) p.38-42 (this paper has a great picture)

The vertebral artery adopts a serpentine course in relationship to the C2 vertebra, making it susceptible to injury during the surgical procedures in the region. The multiple loops of the artery and a buffer space within the vertebral artery groove on the inferior surface of the superior facet of the C2 vertebra and over the posterior arch of atlas provide the artery an extra length and space, probably essential to avoid any stretch during neck movements.

Question 24Which of the following mechanism induces the directional migration of osteoblasts during bone formation?

Active hyperaemia

Angiogenesis

Chemotaxis

Increased microvascular permeability

Hydrostatic pressureReferencesIGF-I secreted by osteoblasts acts as a potent chemotactic factor for osteoblasts.2008 Nov; 43(5): 869-79.Nakasaki M, Yoshioka K, Miyamoto Y, Sasaki T, Yoshikawa H, Itoh K.

IGF-I secreted from osteoblasts in the bone tissue is a potent chemotactic

Page 19: UKITE 2011

factor that may play a major role in recruitment of osteoblasts during bone formation.

Question 25

The following signs all correlate to a cruciate ligament injury EXCEPT

Positive Lachman's test

Positive dial test in 90 degrees flexion

Positive dial test in 30 degrees flexion

Positive pivot shift test

Positive quadriceps active testReferencesThe pivot shift test. Clin Orthop Relat Res. 1976 Jul-Aug;(118):63-9.Use of the quadriceps active test to diagnose posterior cruciate-ligament disruption and measure posterior laxity of the knee. J Bone Joint Surg Am. 1988 Mar;70(3):386-91. Campbell’s operative Orthopaedics, 11th edition – vol II, part XIII, chapter 43 knee injuries.

Question 26

Which of the following statements regarding through knee amputation is true?

Page 20: UKITE 2011

It is primarily indicated in adults as the distal femur provides 70% of the

femoral strengthA through knee amputation is more functionally successful than a below

knee amputationThe patellar tendon is sutured to the remnants of the cruciates in the

intercondylar notchThe hamstrings are attached to the knee joint capsule to facilitate hip flexion

The ITB is attached to the anterior capsule to enhance adductor function in

single limb stance

ReferencesCampbell’s operative Orthopaedics, 11th edition – vol I, part IV, chapter 11 - lower limb amputations

Disarticulation of the knee results in an excellent end-bearing stump. Newer socket designs and prosthetic knee mechanisms that provide swing phase control have eliminated many of the former complaints concerning this level of amputation. Although the benefit of its use in children and young adults has been proved, its use in the elderly and especially in patients with ischemia has been limited in the United States. This is true mainly because the long flaps required instead could allow a more functional, short transtibial amputation in most instances. These long flaps are subject to necrosis in ischemic limbs. A study showed that knee disarticulation is not as successful in posttraumatic amputations, presumably because of the lack of viable musculature in the zone of injury. Nonetheless, knee disarticulation remains a feasible and even desirable level of amputation for some patients.A BKA is the amputation of choice in most patients.The hamstrings can be sutured to aid hip extension.The ITB would aid hip abduction.

Question 27

A 32 year old man presents with a painful right knee. Plain radiographs reveal a lytic lesion in the metaphysis extending to subchondral bone in the distal femur and involving one-third of the femoral diameter. CT scan

Page 21: UKITE 2011

reveals no cortical reaction around the lesion. Which of the following is the best treatment option?

Chemotherapy alone

Intra-lesional currettage and bone grafting

Intra-lesional steroid injection

Radiotherapy alone

Wide local excision and knee reconstruction

ReferencesGiant Cell tumour of bone. The long term results of treatment by currettage and bone graft. Zhen W, Yaotian H, Songjian L, Ge L, Qingliang W. J Bone Joint Surg Br 2004 Mar;86(2):212-6Chapter 9 Orthopaedic Pathology, Review of Orthopaedics 5th Edition, Mark D Miller

The tumour is a GCT- a common benign but locally aggressive tumour. Age 20-50 years. An expansile lytic lesion, involving metaphysis and epiphysis. Usually found in distla femur, proximal tibia and distla radius. Treatment is by intralesional excision and adjunctive measures for stage 1 and 2. Stage 3 and 4 are treated with en bloc excision with approriate reconstruction.

Question 28

Which one of the following is true with regards principles of obtaining a biopsy for a suspected tumour

Adequate haemostasis is not essential

Page 22: UKITE 2011

Biopsy samples negative for neoplasm need not be sent for bacteriological

analysisFrozen section should not be performed routinely on the sample to ensure

adequate tissue has been obtainedIt is preferrable to biospy through muscles rather than an intermuscular tract

The biopsy tract should not be excised

ReferencesReview of orthopaedics, Miller, 5th edition 2008, elsevierhealth publishing, ISBN 9781416040934 p500Adequate haemostasis is important, drains are routinely used. Negativebiopsy samples should be sent for culture and senstivity if there is doubt about infection. Frozen sections should be sent to ensure diagnostic tissue has been obtained. Care should be taken to contaminate as little tissue as possible, deep incision should go through a single muscle compartment rather than through an intermuscular plane. Biopsy track should be considered as a contaminated tract and excised.

Question 29

The rigidity or bending stiffness of a plate is proportional to

The thickness of the plate

The thickness of the plate to the second power

Page 23: UKITE 2011

The thickness of the plate to the third power

The length of the plate

The number of screws

ReferencesThe bending stiffness is proportional to the second moment area, for a rectangle this is proportional to the distance from the neutral axis to the power of three.Therefore doubling the plate thickness increases its bending stiffness eightfold.The torsional stiffness in a cylinder is proportional to the effective radius of the object to the power 4.Review of Orthopaedics, MillerRamachandran Basic Sciences

Question 30

In the process of wound healing, macrophages

Precede the appearance of neutrophils

Are activated by interleukin-2

Page 24: UKITE 2011

Synthesise and secrete tumour necrosis factor

Are derived from megakaryocytes

Are not involved in humoral immunity

ReferencesThere are four phases to wound healing. Haemostasis, inflammation, proliferation and remodellingPlatelets aggregate to allow haemostasis and are fragments of megakaryocytesNeutrophils predominate initally followed by fibroblasts and macrophages (at 2 days)Macrophages are involved in humoral and cell mediated immunity and secrete growth factors and cytokinesIL-2 stimulates T cell proliferation

Question 31

A 15-year old boy presents with back pain. Plain radiographs show vertebra plana at L1. Which is the most likely diagnosis?

Discitis

Eosinophillic granuloma

Osteoblastoma

Scheuermann's disease

Rickets

ReferencesFloman Y, Bar-On E, Moshieff R, Mirovsky Y, Robin GC, Ramu N. Eosiniophillic Granuloma of the Spine. J Pediatr Orthop B 1997 Oct;6(4) 260-5

Question 32

With regards to the Herring classification of Perthes' disease, which of the following is true:

It is based on the degree of medial pillar involvment.

Page 25: UKITE 2011

It is based on the extent of epiphyseal involvement in the reossification

stage.

Group B hips tend to do better in older rather than younger patients.

The classification is based on the appearance in the frog-leg lateral

radiograph of the hips.

Loss of 25% pillar height would classify the hip as group B.

ReferencesHerring classification is based on the degree of flattening of the lateral pillar i.e. the lateral one third of the upper femoral epiphysis as seen (1) on an AP pelvis view (2) in the fragmentation stage.

3 groups described:- Group A: No flattening.- Group B: Upto 50% flattening.- Group C: More than 50% flattening.

Worse the involvement, worse the prognosis. Thus group C hips do worse than group B who do worse than Group A.

In any group, older the patient, worse the outcome. This is due to the decreasing remodelling potential with increasing age.

Review of Orthopaedics, Miller, 5th Edition, pg 229-230

Question 33

Which of the following structures is responsible for medial displacement of the neurovascular bundle in Dupuytren's disease of ring finger?

Central cord

Natatory cord

Page 26: UKITE 2011

Grayson's ligament

Spiral cord

Pretendinous band

ReferencesHayton M.J. and Gray I.C.M. Dupuytren's contracture: a review. Current Orthopaedics 2003; 17: 1-7A spiral cord occurs when four normally existing structures (pretendinous band, spiral band, lateral digital sheet, and the Grayson ligament) become diseased. The spiral cord runs dorsal to the neurovascular bundle proximally and volar to it distally. When the spiral cord is contracted, the neurovascular bundle is drawn toward the midline of the finger. Neurovascular displacement is found most commonly on the ulnar aspect of the little and ring fingers, and tedious dissection is required to prevent digital nerve injury.

Question 34In malignancy with skeletal metastases, which of the following statements is NOT TRUE?

Serum calcium is increased

Serum phosphate is normal or increased

Alkaline Phosphatase is normal or increased

PTH is normal or reduced

Urinary calcium is reduced

ReferencesMillerTable 1-15, p22 of Miller (5th ed), 2nd line: Malig with bony mets. Serum phosphate N or increased. Urinary calcium increased

Medscape / EmedicineHypercalcaemia of malignancy is due to either direct action of tumour cells on bone or osteoclast stimulation (via RANKL) by PTHrP produced by the tumour.It also acts on the kidney to increase calcium reabsorption by the PCT as well as increasing renal phosphate secretion by the kidney. Finally the action of PTHrP (like PTH) increases conversion of 25-OH Vitamin D to 1,25 OH Vit D which increases production of calcium binding protein and therefore gut absorption

Page 27: UKITE 2011

Question 35

A patient with far lateral disc prolapse at L4/L5 is likely to present with

Loss of ankle reflex.

Altered sensation on the dorsum of the foot in the first web space

Weakness of Extensor Hallucis Longus

Foot drop

Weakness of Flexor Hallucis Longus

ReferencesMiller M D: Review of Orthopaedics. Sauders, 2006.

Far lateral disc prolapse at L4/5 affects the exiting L4 nerve root. A paracentral prolapse would affect the traversing L5 root.Ankle jerk - S11st web space of the foot innervation - L5Extensor hallucis longus - L5Flexor Hallucis longus - S1

Question 36

According to the Denis three-column classification, which of the following structure forms part of the posterior column of the spine?

Posterior intervertebral disc

Posterior vertebral body

Posterior ligamentous complex

Posterior longitudinal ligament

Annulus fibrosus

References

Denis F. Spinal instability as defined by the three-column concept in acute spinal trauma. Clin Orthop Rel Res Oct 1984;(189):65-76

The anterior column consists of: anterior 2/3 of vertebtral body, anterior annulus, anterior longitudinal ligament.

Page 28: UKITE 2011

Middle column: posterior wall of vertebral body, posterior annulus, posterior longitudinal ligamentPosterior column: Posterior ligamentous complex connecting neural arches - consisting of facet capsules, ligamentum flavum, interspinous ligament & supraspinous ligament.

Failure of two or more columns generally results in instability.

Don't get confused between the posterior longitudinal ligamentous complex and the posterior spinal ligamentous complex.

Question 37

Meyers & McKeever type III fractures of the tibial spine are best managed by

Arthroscopic / open reduction and internal fixation.

Closed reduction under anaesthesia and casting.

Non weight bearing with crutches and regular observation.

Partial weight bearing with crutches and regular observation.

Aspiration of knee and casting.

ReferencesTachdjian's Pediatric Orthopaedics. 4th Edition, 2008. Lower Extremity Injuries; Knee: pp 2653-2712.

Type III fractures are best treated operatively with an arthroscopic or open technique using suture or a screw (the choice depending upon the surgeon's expertise). The knee is held in full extension for 4 weeks, followed by strengthening and range of motion exercises.

The inter-meniscal ligament keeps the fragment from sitting down in its bed and needs to be fished out of the fracture.

If untreated or inadequately treated, type III fractures can result in ACL deficiency symptoms and signs and can act as a mechanical block to full extension.

Question 38

Page 29: UKITE 2011

A 64-year old lady has presented with a closed distal femoral peri-prosthetic fracture above a closed box PCL sacrificing TKR. She was previously fit, well and independently mobile. Radiographs reveal no loosening of the femoral component. Which of the following would most commonly be used for the management of this fracture?

Constrained revision TKR

Hinged revision TKR

Retrograde supracondylar femoral intramedullary nailing

Distal femoral locking plate osteosynthesis

Circular frame external fixation

References

J Am Acad Orthop Surg, Vol 12, No 1, January/February 2004, 12-20. © 2004 the American Academy of Orthopaedic Surgeons

The Journal of Bone and Joint Surgery (American) 83:120 (2001)2001Instructional Course Lecture Periprosthetic Fractures Following Total Knee Arthroplasty Douglas A. Dennis, MD

The Journal of ArthroplastyVolume 20, Supplement 2, June 2005, Pages 27-32 Periprosthetic Fractures After Total Knee Arthroplasty Ravi Tharani MD, Cass Nakasone MD and Kelly G. Vince MD,

the “Less Invasive Stabilization System” (LISS), has been used for supracondylar fractures above TKAs. Althausen et al reviewed results from different treatment methods for supracondylar femur fractures above TKA and noted advantages to the LISS plate, characterized by threaded screw holes in the plate that maintain the screws solidly in the plate. This increases fracture stability especially in osteoporotic bone. This technique is specifically helpful where a retrograde nail cannot be passed due to a closed box TKR. A small incision is possible as the screws can be placed percutaneously. This potentially reduces the need for bone graft, risk of infection, and blood loss. Patients can usually be mobilized soon after surgery to avoid complications of immobilization.

Page 30: UKITE 2011

Kregor et al had favorable results in 13 supracondylar femur fractures above TKA in 11 patients treated with LISS plating.

Question 39

What is the % energy expenditure above baseline during ambulation following an average transtibial amputation?

10%

25%

45%

65%

75%

ReferencesMiller, review of orthopaedics, 5th edition - chapter 10, part II

Transtibial (below-knee) amputation—A long posterior myocutaneous flap is the preferred method of creating a soft tissue envelope. The optimum bone length is at least 12 cm below the knee joint or longer if adequate gastrocnemius or soleus muscle can be used to construct a durable soft tissue envelope. Posterior muscle should be secured to the beveled anterior tibia by myodesis. Rigid dressings are preferred during the early postoperative period, and early prosthetic fitting may be started 5-21 days after surgery if the residual limb is capable of transferring load and the patient has a satisfactory physical reserve.

Amputation Level % Energy Above BaselineLong transtibial 10Average transtibial 25Short transtibial 40Bilateral transtibial 41Transfemoral 65Wheelchair 0-8

Question 40

Page 31: UKITE 2011

In which of the following scenarios would thoracic back pain NOT be a common finding?

Thoracic spinal stenosis

Thoracic compression fracture

Thoracic discitis

Adolescent idiopathic thoracic scoliosis

Thoracic costovertebral arthritis

ReferencesSales et al 'Osteoarthritis of the costovertebral joints' JBJS (Br) 2007;89-B:1336-9

IDIOPATHIC adolescent scoliosis by and large does not cause back pain. In fact presence of back pain in scoliosis should prompt further investigation eg MRI to exclude an underlying disorder before being labelled as idiopathic.

Question 41In the posterolateral corner of knee, which of the following structures is not part of the complex?

Popliteus tendon

Fabellofibular ligament

Popliteofibular ligament

Patellofemoral ligament

Lateral Gastrocnemius tendon

ReferencesThe patello-femoral ligament is found on the medial side of the knee.

Question 42

A reported disadvantage of osteochondral allografting is

Longer operative time

Longer graft incorporation period

Availability of larger and increased number of grafts

Donor site morbidity

Page 32: UKITE 2011

Potentially lower incidence of arthrofibrosis

References

Suarez,Luis Sierra MD; Richmond, John C. MDOverview of procurement, processing and sterilisation of soft tissue allografts for sports medicineSports Medicine and Arthroscopy. Volume 15(3), September 2007, pp 106-113

The clinical and surgical advantages of allograft are well desribed and are as follows:availability to surgeon, precise preperation of graft in any size and shape, lack of donor site morbidity, shorter operative time and lack of clinically significant immunological reaction. Disadvantages are: potential for disease transmission, immunological reaction with graft rejection, cost, higher incidence of arthrofibrosis, limited availability and demanding surgical technique.

Question 43

Which of the following is the predominant blood supply to the anterior cruciate ligament

Popliteal artery

Superior medial genicular artery

Inferior lateral genicular artery

Middle genicular artery

Superior lateral genicular artery

Referenceswww.wheelessonline.com/ortho/anatomy_of_acl

Question 44

The number of separate osteofascial compartments in the hand are:

5

Page 33: UKITE 2011

7

8

10

12

ReferencesGreens Text book of Hands

Wheelers orthopaedics

10 separate osteofascial compartments which typically can be released w/ carpal tunnel release and 1 or 2 dorsal incisions; - dorsal interossei (4 compartments) - palmar interossei (3 compartments) - adductor pollicis - thenar and hypothenar

Question 45

Which one of the following options is an absolute indication for amputation in a Gustillo IIIc open tibial fracture, according to Lange?

Cold ischaemic time exceeding 6 hours

Complete posterior tibial nerve disruption

Associated polytrauma

Severe ipsilateral foot trauma

Fracture with major soft tissue and bony reconstruction anticipated

ReferencesKeene GS, Robinson AHN, Bowditch MG, Edwards DJ. Key topics in orthopaedic trauma surgery. BIOS scientific publishers limited, 1999; 3-5

Lange RH, Bach AW, Hansen ST Jr, Johansen KH. Open tibial fractures with associated vascular injuries: Prognosis for limb salvage. J Trauma 1985;25:203-8.

Lange et al., proposed a decision-making protocol based on absolute and relative indications. The occurrence of just one of two absolute indications

Page 34: UKITE 2011

(complete posterior tibial nerve disruption in adults; crush injuries with longer than six hours of warm ischemia time) warrants primary amputation, while at least two of three relative indications (serious associated polytrauma, severe ipsilateral foot trauma or projected long course to full recovery) must be present to reach that decision

Question 46Which is NOT a recognised method of treating tibial non union

Exchange nailing

Dynamisation

Bisphosphonates

Pulsed electromagnetic stimulation

Open reduction and internal fixation with bone graft

ReferencesCourt-Brown, et al. Exchange intra-medullary nailing. JBJS 1995;77B:407-411Rockwood and Green's. Fractures in Adults

Question 47

Leadbetter’s manoeuvre for the closed reduction of intracapsular proximal femoral fractures involves:

Flexion and abduction, traction, external rotation, adduction, extension.

Flexion and abduction, traction, internal rotation, adduction, extension.

Flexion and adduction, traction, internal rotation, abduction, extension.

Flexion and adduction, traction, external rotation, abduction, extension.

Page 35: UKITE 2011

Flexion and adduction, external rotation, traction, internal rotation,

extension.

ReferencesLeadbetter's 1933 and 1938 papers (Both JBJS-Am)

Question 48

A 50 year old male presents with a painful swelling of his right ankle. MRI demonstrates that it is arising from the soft tissues and biopsy confirms the diagnosis of malignancy. The histopathological report is most likely to show which of the following?

Kaposi sarcoma

Giant cell tumour of the sheath

Malignant Fibrous Histiocytoma

Synovial Sarcoma

Rhabdomyosarcoma

ReferencesSoft-tissue tumors and tumor-like lesions of the foot. An analysis of eighty-three cases. EJ Kirby, MJ Shereff and MM Lewis J Bone Joint Surg Am. 1989;71:621-626.Synovial sarcoma is the commonest malignancy in the foot. GCT is commoner but is benign

Question 49

A 45-year old man presents with an acutely painful knee and is correctly diagnosed with Calcium Pyrophosphate Dehydrate Deposition (CPPD). Which one of the following is true?

Surgical washout is often required

Is diagnosed by the presence of positively birefringent crystals

Does not commonly affect the knee

Page 36: UKITE 2011

NSAIDS are ineffective

Affects 10 x more men than women

References

O'Duffy, JD. Clinical studies of acute pseudogout attacks. Comments on prevalence, predispositions, and treatment. Arthritis & Rheumatism; 1976. Supplement. (19): 349–352.male: female 2:1 in most studies.NSAIDS usually effective pain relief and surgical washout is not usually required. Imaging is useful in confirming the diagnosis, but does not differentiate between gout and pseudogout. Sometimes there is evidence of chondrocalcinosis on the radiograph.Positive birefringence is diagnostic.

Question 50

With regards to high pressure injection injuries, which substance when injected has the highest risk of requiring amputation?

Oil

Paint

Grease

Fuel

Air

References

Wheeless onlineemedicineHigh pressure grease and paint guns are most common cause of injury, & site of injury is often the pad od the thumb or index finger; - severity of injury is often underestimated due to the small punctate entrance wound; - injected substance passes rapidly thru the subcutaneous tissue and enters the flexor tendon sheath; - from there it passes into one or more of the deep spaces of the hand

Page 37: UKITE 2011

- incidence: paint: 60% vs grease: 25% - patient profile: - young male; - new job; - non dominant hand (75%) - prognostic factors: - material injected: - grease (fibrosis) - paint (necrosis) : paint causes an immediate tissue necrosis that persists if the tissues are not completely debrided; - pressure: - < 7,000 psi - non prognostic - > 7,000 psi - 100% amputation

Question 51

What is the most likely cause of persistent lateral ankle pain following a snowboarding injury not responding to conservative treatment?

Subtalar dislocation

Peroneal tendon subluxation

Cuboid fracture

Fracture of the lateral process of Talus

Fibular stress fracture

ReferencesFractures of the Talus. Rockwood & Green's Fractures in the Adult. 6th Edition. Question 52

Vertebral disc infection in children is most commonly caused by which pathogen?

Staphylococcus epidermidis

Staphylococcus aureus

Escherichia Coli

Pseudomonas aeruginosa

Legionella pneumophilia

Page 38: UKITE 2011

ReferencesReview of Orthopaedics. Miller MD Fifth edition: P491.S. aureus is most common pathogen in children - usually via haematogenous spread since vertebral end-plates are still open.

Question 53

A 59 year old man has a 3 year history of increased pain in his right foot with deformity. Examination reveals hindfoot fixed valgus of 15 degrees with midfoot abduction. He has tried nonsurgical treatment without relief.

Which of the following would be the most appropriate surgical management?

Medial sliding os calcis osteotomy

Subtalar arthrodesis

Talonavicular arthrodesis

Subtalar arthrodesis with FDL transfer

Triple arthrodesis

ReferencesCore Knowledge in Orthopaedics. Foot and Ankle.A fixed planovalgus foot is a Johnson and Strom grade 3 deformity which requires triple arthrodesis

Question 54

Which one of the following statements regarding Achondroplasia is FALSE?

It is the most common form of disproportionate dwarfism

The genetic mutation is in the Fibroblast Growth Factor Receptor 3 (FGFR3)

gene

Scoliosis is a common feature.

It is strongly related to paternal age

Page 39: UKITE 2011

Narrow interpedicular distances can lead to development of a significant

spinal stenosis

ReferencesOxford Textbook of Orthopaedics and Traumapg 47-48

Thoracolumbar kyphosis is commonly seen, and often resolves at the time of ambulation. Patients often develop excessive lordosis due to structural abnormalities in the lumbar vertebrae.

Question 55

A young male manual worker presents with secondary arthritis of the index finger PIPJ of his dominant hand. With regard to arthroplasty, which of the following is true:

Silicon arthroplasty has good long term resultsPoor flexion range may follow use of an undersized silicon arthroplasty

component

Pyrocarbon arthroplasty is not associated with squeaking implants

Arthrodesis may be better than arthroplasty if he wishes to remain in work

Apparent loosening of a pyrocarbon implant on radiographs is always due to

true implant looseningReferencesThe Journal of Hand SurgeryVolume 35, Issue 12, December 2010, Pages 2107-2116

Question 56A patient, who underwent in-situ pinning for SUFE and was asymptomatic immediately following surgery but has subsequently developed pain, stiffness and flexion deformity. Which of the following is the most likely diagnosis

AVN

Page 40: UKITE 2011

Chondrolysis

Further slip

Normal

Pin penetration ReferencesAlthough rare, chondrolysis is one of the recognised complication of SUFE. It usually presents as pain and significant restriction of the joint movement. Tachdjian: Pediatric orthopaedics Second Edition Vol 2, Page no 1066

Question 57A displaced fracture of the talar neck with dislocation of the body of the talus (ankle joint) but NOT the head (talonavicular joint) would be classified as:

Hawkins I

Hawkins II

Hawkins III

Hawkins IV

Hawkins V

ReferencesKoval KJ, Zuckerman JD. Handbook of fractures. Chapter 40 p.437

Hawkins classification is important as it correlates well with the risk of subsequent talar avascular necrosis.

Question 58

In Scapholunate advanced collapse (SLAC) wrist which area is classically NOT involved

Scaphocapitate Joint

Radial Styloid

Radiolunate joint

Radioscaphoid Joint

Capitolunate Joint

References

Page 41: UKITE 2011

Millerscapholunate advanced collapse (SLAC) refers to a specific pattern of osteoarthritis and subluxation which results from untreated chronic scapholunate dissociation or from chronic scaphoid non-union;- degenerative changes occur most often in areas of abnormal loading;- radial-scaphoid joint is involved initially, followed by degeneration in the unstable lunatocapitate joint, as capitate subluxates dorsally on lunate;

- Radiographs:- radioscaphoid joint is first to develop degenerative changes;- capitolunate & STT joints, follow in order w/ degenerative changes;- capitate migrates proximally into space created by scapholunate dissociation;- radiolunate joint is usually spared because of concentric articulation of lunate w/ in speroid lunate fossa of distal radius;- lunate:- w/ end stage SLAC midcarpal joint collapses under compression & lunate assuming an extended or dorsiflexed position (DISI deformity);

Question 59

Regarding subtalar dislocation, which one of the following statements is true

Lateral dislocation is more commonMedial dislocation requires open reduction more frequently than lateral

dislocationSuccessful closed reduction of lateral dislocation can be prevented by

interposed tibialis posterior tendon.

Approximately 10% of lateral dislocations require open reduction

Is usually unstable once reduced

References

Page 42: UKITE 2011

http://www.wheelessonline.com/ortho/sub_talar_dislocation

Lateral dislocation is a less common type of subtalar dislocation (15%). It occurs when calcaneus is displaced lateral to talus; Talar head lies medially, and foot appears pronated; Navicular lies lateral to the talar neck; lateral dislocations may be complicated by interposed posterior tibial tendon (or sometimes FDL); approx 20% of lateral dislocations require open reduction; incision over sinus tarsi, and three wks of NWB casting, followed by ROM;

Medial dislocation, is the most common sub talar dislocation (85%); foot & calcaneus are displaced medially; head of the talus prominent dorsolaterally; navicular lies medial and sometimes dorsal to talar head & neck; foot is plantar flexed and is supinated; inversion causes this injury; approx of 10% of medial dislocations require open reduction; capsule of talonaviclar joint & EDB blocks reduction, or in some cases the talar head may button hole thru the EDB; medial sub talar dislocations are treated by longitudinal anteromedial incision over prominent head and neck of talus & minipulation and release of interposed tissues; since the joint is stable after reduction there is no need for internal fixation; after reduction, a short leg cast is applied for 3 to 4 wks;

Question 60

A 40 year old man presents to your arthroplasty clinic with a painful right hip. A plain radiograph reveals a dysplastic acetabulum with an 80% subluxed femoral head. How should the dysplasia be classified?

Howie Classification A2 F3a

Crowe Grade 2

Crowe Grade 3

Crowe Grade 4

Howie Classification A3b F3b

References

Page 43: UKITE 2011

Review of orthopaedics. 4th ed. Miller M.D.grade 1: hips have less than 50% subluxation; grade 2: hips have between 50% to 75% subluxation; grade 3: hips have between 75% to 100% subluxation;grade 4: hips have more than 100% subluxation;

Howie classification (Hip International, 2009)The acetabular classification comprises: AI: Dysplastic acetabulum; AII: The acetabulum associated with a low femoral dislocation; AIII: The post-surgical acetabulum, with (AIIIa) or without retained metalwork (AIIIb). The femoral classification consists of: FI: Dysplastic femur but contained within true or low acetabulum; FII: The high femur; FIII: Post-surgical femur, again with or without metalwork (FIIIa and FIIIb).

Question 61

Which of the hindfoot joints has the highest incidence of non-union following a triple arthrodesis?

Calcaneocuboid

Talonavicular

Subtalar

Ankle

Calcaneonavicular

ReferencesMizel MS, Sobel M: Disorders of the Foot and Ankle. In Miller MD (ed): Review of Orthopaedics, 2nd Ed. Philadelphia. W.B. Saunders, 1996, pp223-243.

Isolated talonavicular fusion is associated with a high non-union rate. This is thought to be because its movement is closely linked to that of the subtalar joint. If the subtalar joint remains mobile an excess load is placed through the talonavicular joint. Furthermore the navicular has a poor blood supply.

Question 62

Which of the following is NOT a recognised technique for evaluating postero-lateral corner injuries?

Page 44: UKITE 2011

Increased external rotation

Postero-lateral drawer test

Reverse pivot shift test

Dial test at 15º and 60º

External rotation recurvatum test

ReferencesSekiya & Miller et al. A clinically relevant assessment of posterior cruciate ligament and posterolateral corner injuries. J Bone Joint Surgery (Am) 2008; 90: 1621-7.

MD Miller. Review of Orthopaedics. Fourth edition. 2004. Saunders Elsevier. Page218

Posterolateral corner injuries are rarely isolated injuries and are commonly associated with other ligament injuries, especially the PCL. Examination for increased external rotation, external rotation recurvatum test and, posterolateral drawer test and reverse pivot shift test are important. The Dial test should be performed at prone/supine (dial test) 30/90 degrees knee flexion, with ER torque and 10 degree difference compared to contralateral leg is a positive test.

Question 63

Advanced Trauma Life Support recommendations state that adult patients with second or third degree burns should be resuscitated with Ringer’s Lactate solution according to the following formula:

1 to 2 ml per kg body weight per percent burn in the first 24 hours

2 to 4 ml per kg body weight per percent burn in the first 24 hours

8 to 10 ml per kg body weight per percent burn in the first 24 hours

10 to 12 ml per kg body weight per percent burn in the first 24 hours

No specific guidelines for adults

References

Page 45: UKITE 2011

ATLS Student Course Manual Fluid resuscitation requirements in burns patients are calculated using the Parkland formula which advocates resuscitation with crystalloid over the first 24 hours with 2-4 mls/ kilogram of body weight per percentage burn of Total Body Surface Area (TBSA). Half the volume is given in the first 8 hours post burn, with the remaining volume delivered over 16 hours. The aim is to produce a urine output in adults of 0.5-1.0mls/ kg/ hour.

Question 64Local anaesthetic agents are less effective in the presence of infection due to which one of the following options

A higher ionised concentration of the agent

A lower ionised concentration of the agent

An alkaline pH in the tissue fluid

Local vasodilatation

Micro-organisms ingesting the local anaesthetic agent

ReferencesOnly the non-ionised form of local anaesthetics (LA) can traverse the neuronal membrane, in order to access the sodium channels from inside the cell. The acidic environment, caused by the presence of infection, results in a higher concentration of ionised LA. This produces relative inactivity of the LA.

Question 65The essential fluoroscopic views for percutaneous screw fixation of sacro-iliac joint would be

Pelvis AP, inlet, obturator outlet and lateral sacral.

Pelvis AP, iliac inlet, outlet and lateral sacral.

Pelvis AP, iliac inlet, obturator outlet and lateral sacral.

Pelvis AP, inlet, outlet and lateral sacral.

Pelvis AP, iliac inlet, obturator outlet and “teepee”.

References

Hilgert RE, Finn J, Egbers HJ. Technique for percutaneous iliosacral screw insertion with conventional C-arm radiography. [Article in German].

Page 46: UKITE 2011

Unfallchirurg. 2005 Nov;108(11):954, 956-60.Rockwood and Green's Fracture in Adults 6th rev edn (1 Dec 2005) Lippincott Williams & Wilkins. Editors Robert W. Bucholz, James D. Heckman, Charles M. Court-Brown, and Paul Tornetta. Chapter 41 Fracture of the Pelvic Ring. pp 1583-1664

Question 66

The following are distal realignment procedures of the patella EXCEPT

Elmslie-Trillat procedure

Maquet procedure

Insall procedure

Fulkerson osteotomy

Hauser procedure

Referenceswww.wheelessonline.com/ortho/distal_realignment_for_patellar_subluxationhttp://www.wheelessonline.com/ortho/insall_proximal_realignment_for_disorders_of_the_patella

Page 47: UKITE 2011

Fulkerson Osteotomy: - modification of the Elmslie-Trillat Procedure, but involves anterior displacement as well; - main indications are persistent pain and moderate articular degeneration; - allows anteriorization of upto 15 mm, which should decrease lateral facet contact pressure;

Hauser Procedure: - discussed for historical purposes only; - involves medialization of the tibial tubercle inorder to decrease Q angle; - due to the anatomy of the proximal tibia, translating the tibial tubercle medially, will also translate the tubercle posteriorly; posterior translation of the tibial tubercle will have the effect of increasing patellofemoral contact pressures which leads to DJD;

Maquet Procedure: - discussed for historical purposes only; involves anterior translation of the tibial tubercle which has the effect of decreasing patellofemoral contact forces; patients w/ pain due to early patellofemoral arthrosis may expect pain relief following the Maquet Procedure; -disadvantages w/ this procedure include high incidence of skin necrosis, and no effect on the Q angle;

Elmslie-Trillat Procedure: - medial tibial tubercle transfer which has no posterior displacement; - does not involve anterior displacement of the tuberosity;Insall procedure: Proximal realignment with standard lateral retinacular release.

Question 67

In the surgical treatment of de Quervain's stenosing tenosynovitis, which of the following tendons should be decompressed?

Abductor Pollicis Longus

Adductor Pollicis

Extensor Pollicis Longus

Flexor Pollicis Longus

Opponens Pollicis

References

Page 48: UKITE 2011

Campbell's Textbook of Operative Orthopaedics, 10th Ed, Part XVIII The Hand, page 3772 "Stenosing Tenosynovitis".It is named after the Swiss surgeon Fritz de Quervain who first identified it in 1895.

The two tendons concerned are the tendons of the extensor pollicis brevis and abductor pollicis longus muscles. The tendons run, as do all of the tendons passing the wrist, in synovial sheaths, which contain them and allow them to exercise their function whatever the position of the wrist. Evaluation of histological specimens shows a thickening and myxoid degeneration consistent with a chronic degenerative process The pathology is identical in de Quervain seen in new mothers.De Quervain's is more common in women; the speculative rationale for this is that women have a greater angle of the styloid process of the radius.Symptoms are pain, tenderness, and swelling over the thumb side of the wrist, and difficulty gripping.Finkelstein's test is used to diagnose de Quervain syndrome in people who have wrist pain. To perform the test, the examining physician grasps the thumb and the hand is ulnar deviated sharply, as shown in the image. If sharp pain occurs along the distal radius DeQuervain's syndrome is likely.

Question 68

A patient presents with midfoot pain and swelling after falling down stairs.The following finding on plain radiograph does NOT imply Lisfranc joint injury:

Avulsion fracture of the second metatarsal base

Dorsal displacement of the first metatarsal baseMedial edge of the second metatarsal base aligned with medial edge of

middle cuneiformMedial edge of fourth metatarsal base aligned with medial edge of lateral

cuneiform

Plantar displacement of second metatarsal base

ReferencesRockwood & Green's Fractures in Adults (3rd edition): Injuries of the tarsometatarsal joints, p.2145

Page 49: UKITE 2011

Question 69

When positioning a patient with an extracapsular fracture of the neck of femur on the fracture table, which one of the following will most accurately help you assess the rotation of the distal femur.

The rotation of the foot in the holding device

The position of the patella

The abduction angle of the leg

The greater trochanter

The appearance of the femoral head on the image intensitier

References

Question 70

Radiographic appearance of hyperostosis resembling "molten wax" running down the cortex of bone is characteristic of

Rickets

Melorheostosis

Osteopetrosis

Osteogenesis imperfecta

Fibrous dysplasia

ReferencesPriniciples of Orthopaedic Practice. Roger Dee. Second edition. P-678

Question 71

Page 50: UKITE 2011

During fracture healing, differentiation of the progenitor cells depends on local oxygen tension and strain conditions. Based on this theory, which one of the following promotes formation of woven bone during fracture healing.

High strain and low oxygen tension

Low strain and high oxygen tension

Intermediate strain

Low oxygen tension

Intermediate strain and low oxygen tension

ReferencesMiller, Review of Orthopaedics (5th edition) - Page 12

Question 72

Which of the following options is NOT associated with rickets

Phosphate diabetes

Lack of dietary oxalate

Organ insensitivity to autogenous 1,25-dihydroxyvitamin D production

Free fatty acids in intestinal system

Neurofibromatosis

Referencespediatric orthopaedicscore knowledge in orthopaedicsjohn dormanselsevier, mosby

Page 51: UKITE 2011

Vitamin D-resistant rickets is also called as phosphate diabetes.Neurofibromatosis can cause tumors secreting putative factors and subsequent renal osteodystrophyFree fatty acids in intestinal system could be due to biliary disease with interference with absorption of fat-soluble vitamin DIncrease in dietary oxalate and phytates can chelate calcium and lead nutritional ricketsEnteric end-organ insensitivity to 1,25 dihydroxyvitamin D leads to Vitamin D-dependent rickets, type II

Question 73

Massive blood transfusion implies a transfusion ratio of - packed red blood cell:fresh frozen plasma:platelet

3:2:1

5:1:1

1:1:1

6:2:1

3:1:1

ReferencesInjury. 2010 Jan;41(1):35-9.Massive transfusion is arbitrarily definied as the replacement of a patient's total blood volume in less than 24 hours, or as the acute administration of more than half the patient's estimated blood volume per hour.

Page 52: UKITE 2011

Question 74

Regarding the synovium, the following statement is true

It resembles endothelium

It is lined by stratified epithelium

Type A synovial cells resemble macrophages

It is exquisitely sensitive to pain

It has poor lymphatic drainage

References

Synovium has good vascular and lymphatic supply. It has good nerve supply but it is not very sensitive to pain. Type B synovial cells resemble fibroblasts. Synovium has no epithelial component.

Question 75

Which one of the following is true with regards to the technique of combining lag screw fixation with tension band wiring of a patella fracture Results in a construct with greater load to failure than either lag screw

fixation or tension band wiring alone. Results in a construct with lower load to failure than either lag screw fixation

or tension band wiring alone. Results in a construct with equal load to failure than either lag screw fixation

or tension band wiring alone.

Does not allow early knee mobilisation

Should not be undertaken for transverse fractures

ReferencesReview of orthopedic Trauma . M R Brinker

Page 53: UKITE 2011

Question 76

Which of the following options is FALSE with regards to Tuberculous arthritis?

Juxta-articular osteoporosis

A chronic historyHigher concentrations of proteolytic enzymes in joint fluid as compared to

pyogenic arthritis

More likely to result in fibrous ankylosis than pyogenic arthritis

Involvement of only one joint

References"Apley's System of Orthopaedics and Fractures" Louis SolomonDavid WarwickSelvadurai Nayagam

Question 77

The following is true about errors in clinical trials

Type I error is incorrectly accepting the null hypothesis

Type II error is incorrectly rejecting the null hypothesis

Type III error does not exist

We can protect against a type II error with statistical power analysis

Type I error can be reduced by increasing the significance levels

ReferencesBasic Orthopaedic Sciences - The Stanmore Guide.Type I error occurs when the null hypothesis is incorrectly rejected.Type II error occurs when the null hypothesis is incorrectly accepted.Type III error occurs rarely when the researcher correctly rejects the null hypothesis but incorrectly attributes the cause.

Page 54: UKITE 2011

Type II error can be reduced with statistical powerType I error can be reduced by reducing the significance levels

Question 78

Which of the following is NOT correct about Paget's disease?

Remodelling disease caused by excessive osteoclastic activity

Common in the fifth decade of life

Common sites include hands and feet

Can cause nerve compression

Bisphosphonate therapy is the mainstay of treatment

ReferencesRosier RN, Bukata SV. Bone metabolism and metabolic done diseases. In: Orthopaedic Knowledge Update 7. Rosemont, Ill: American Academy of Orthopaedic Surgeons; 2002:152-154.

Question 79

Regarding plating of tibial shaft fractures, which of the following statements is INCORRECTInfection is less common in closed fractures treated after a two-week delay.

Delayed union is more common in closed fractures treated after a two-week

delayComminuted fractures are more likely than torsional fractures to be

complicated by non-unionComminuted fractures are more likely than torsional fractures to be

complicated by infection Open fractures are more likely than closed fractures to be complicated by

non-union

ReferencesCampbells, Chapter 51

Page 55: UKITE 2011

Question 80

The AO principles of fracture care includes all EXCEPT

Anatomic reduction of the fracture fragments

Stable internal fixation

Non weight-bearing until radiological fracture union

Preservation of blood supply

Early active pain-free mobilisation

ReferencesBrinker: Review of Orthopaedic Trauma

Question 81

ECG changes that may be seen in Pulmonary Embolism are:

Left bundle branch blockLarge S wave in lead I, a large Q wave in lead III and an inverted T wave in

lead III

Large S wave in lead I, small Q wave in lead III

Normal S wave in lead I, Absent Q wave in lead III,

Complete heart block

ReferencesThe S1Q3T3 sign:• Many call this pattern the “right heart strain pattern”, but the more appropriate term is: acute cor pulmonale• The S1Q3T3 is the ECG manifestation of acute pressure and volume overload of the right ventricle - An S wave in lead I signifies a complete or more often incomplete RBBB

Page 56: UKITE 2011

- In lead III, look for a Q wave, slight ST elevation, and an inverted T wave. These findings are due to the pressure and volume overload over the right ventricle which causes repolarization abnormalities. • The S1Q3T3 was first described by McGinn and White in JAMA in 1935.

Question 82

In Developmental Dysplasia of the Hip (DDH), which of the following statements is true regarding the Ortolani's test?It is a provocative test for DDH that attempts to subluxate the femoral head.

It is often positive in the walking child.It consists of pushing down the greater trochanter while simultaneously

adducting the hip.It consists of lifting up the greater trochanter while simultaneously abducting

the hip.

It is always positive in the infant with DDH.

ReferencesTachdjian's Pediatric Orthopaedics. 4th Edition. 2008. Developmental Dysplasia of the Hip: pp 637-770.

Barlow's and Ortolani's tests are used to diagnose DDH in neonates and infants.

Barlow's is the provocative test for the dislocatable hip and Ortolani is the relocating test for the already 'out' hip. (Remember Ortolani for Out !)

The 'clunk' of Ortolani is a sensation of relocation of the head that is seen and/or felt but not heard. Nevertheless the 'clicky' baby hip always needs to be further investigated via ultrasonography.

Question 83

The most common sequelae of undiagnosed compartment syndrome in the foot is

Page 57: UKITE 2011

Paresthesia in the toes

Clawing of the toes

Chronically swollen foot

Absent pulses

Reflex sympathetic dystrophy

ReferencesMiller orthopaedics

Question 84

Prophylactic fixation is necessary in the management of metastatic bone disease in which of the following options:An osteoblastic lesion in the proximal humerus involving 50% of cortex,

moderate amount of painA mixed lesion in the shaft of humerus, involving 50% of the cortex, mild

amount of painA lytic lesion affecting the shaft of femur affecting 30% of the cortex, mild

pain A mixed lesion affecting the shaft of radius involving 30% of the corttex,

mild painA lytic lesion in the neck of femur involving 50% of the cortex, mild pain

Referenceshttp://www.wheelessonline.com/ortho/pathologic_fractureMain surgical treatment for pathological fractures is IM nailing. Adjuct

Page 58: UKITE 2011

treatment is not usually needed unless there is a very large defect present. According to Mirels' classification, the final option gives a Mirel score of 9, even assuming the minimum score for pai, this requires prophylactic fixation.

Question 85

Following are the possible causes of a false-negative pivot shift test EXCEPT

Medial meniscus tear

Lateral meniscal tear

Medial collateral ligament rupture

Lateral collateral ligament rupture

Iliotibial band rupture

ReferencesInsall J. Surgery of the Knee 3rd edition. Magee. Orthopaedic Clinical Examination.

Question 86

Which of the following investigations best distinguishes osteoporosis from osteomalacia?

Iliac Crest Bone biopsy

Serum Calcium

Serum Phosphate

Urinary Calcium

Tetracycline labelled Bone Biopsy

ReferencesBullough PG, Bansal M, DiCarlo EF: The tissue diagnosis of metabolic bone disease: Role of histomorphometry. Orthop Clin North Am 1990;21:65–79.

Osteoporosis typically presents with normal serum calcium, phosphorus, alkaline phosphatase, vitamin D, and PTH levels. In contrast, osteomalacia is characterised by hypophosphataemia, hypocalcaemia, increased alkaline phosphatase levels, low levels of vitamin D metabolites, and secondary

Page 59: UKITE 2011

hyperparathyroidism.

Urinary calcium levels may be normal in osteoporosis but are low in osteomalacia. Both conditions appear as low bone mass on radiological studies and DXA scan. However, specific radiological findings unique to osteomalacia include Looser pseudofractures.

A bone biopsy is the best differentiating test.

Question 87

With regards to diagnosis of septic arthritis, the threshold of white blood cell (WBC) concentration and proportion of polymorphs (PMN) in the joint aspirate is?

<200 WBCs and 25% PMNs

200 - 2,000 WBCs and 50% PMNs

>50,000 WBCs and 50% PMNs

>80,000 WBCs and 75% PMNs

>200,000 WBCs and 50% PMNsReferences

The Arthroses - Miller, review of orthopaedics, 5th edition – Chapter 1, Section 2/II

Joint fluid analysis1.Noninflammatory arthritides—200 white blood cells (WBCs) with 25% polymorphonuclear neutrophils (PMNs); equal serum values of glucose and protein; normal viscosity (high), straw color, firm mucin clot.2.Inflammatory arthritides—2000-75,000 WBCs with up to 50% PMNs; moderately decreased glucose (25 mg/dL lower than serum glucose); low viscosity, yellow-green, friable mucin clot. Synovial fluid complement is decreased in RA and normal in ankylosing spondylitis.3.Infectious arthritides—More than 80,000 WBCs with more than 75% PMNs, a positive Gram stain (also positive cultures later), low glucose (25 mg/dL lower than serum glucose), opaque fluid, increased synovial lactate.

Question 88

Page 60: UKITE 2011

With regards to the gait cycle, double-limb stance consists of approximately what total percentage of one cycle (stride)

10%

12%

16%

20%

25%

ReferencesOne gait cycle (stride) is defined from initial contact (heel strike) of one heel until the next heel strike of the ipsilateral heel. It can be divided into a stance phase and a swing phase. At the beginning and end of the stance phase there is a period of double-limb support, each consisting of approximately 10% of one total cycle (2 x 10% = 20%). Miller Review of Orthopaedics 5th edition, p553.

Question 89

With regards to the anterior approach (Smith-Peterson) to the hip, which of the following statements is true?

The superficial dissection is between gracilis and sartorius The internervous plane is between the obturator nerve and the femoral nerve

The medial femoral circumflex artery is a common structure at riskThe internervous plane is between the femoral nerve and the superior gluteal

nerve

The superficial dissection is between rectus femoris and sartorius

ReferencesThis approach takes advantage of the interneural plane between the sartorius (femoral nerve) and tensor fasciae latae (superior gluteal nerve). It is useful for operative procedures such as open reduction of the congenitally dislocated hip. The lateral femoral cutaneous nerve is retracted anteriorly, and the ascending branch of the lateral femoral circumflex artery (which lies superficial to the rectus) is ligated. For deeper dissection, approach the

Page 61: UKITE 2011

interval between the gluteus medius and rectus femoris. Detach the origin of both heads of the rectus femoris. Reflection of the conjoined rectus tendon too distally can risk injury to the descending branch of the lateral femoral circumflex artery. Retract the rectus medially and the gluteus medius laterally. Dissect any attachments of the iliopsoas to the inferior capsule and perform a capsulotomy. There is a risk to the lateral femoral cutaneous nerve, which is located anterior or medial to the sartorius about 6-8 cm below the ASIS. The superficial circumflex artery penetrates the tensor fasciae latae just anterior to the lateral femoral cutaneous nerve. The femoral nerve and vessels can sometimes be injured by aggressive medial retraction of the sartorius.

Millers review of Orthopaedics, 5th ed., pp176-7

Question 90

The Segond fracture is associated with an injury to the anterior cruciate ligament and is a result of an avulsion of the

oblique popliteal ligament

popliteus tendon

lateral collateral ligament

lateral capsule

arcuate ligament

ReferencesThe Segond fracture, or avulsion fracture of the lateral capsule, is pathognomonic of an anterior cruciate ligament tear. (Campbells operative orthopaedics, Vol III)Originally described by Dr. Paul Segond (French) in 1879 after a series of cadaveric experiments, the Segond fracture occurs in association with tears of the anterior cruciate ligament (ACL) - Segond P. Recherches cliniques et expérimentales sur les épanchements sanguins du genou par entorse. Progres Med 1879; 7:297-299, 319–321, 340–341.

Page 62: UKITE 2011

Question 91

What muscles are responsible for the most common residual deformity after antegrade intramedullary nailing for a subtrochanteric femoral fracture?

Hip abductors and iliopsoas muscle

Hip Internal rotators and iliopsoas muscle

Quadriceps and iliopsoas muscle

Hamstring and iliopsoas muscle

Quadriceps and hip adductors

References1. French BG, Tornetta P III. Use of an interlocked cephalomedullary nail for subtrochanteric fracture stabilization. Clin Orthop. 1998; 348: 95-100 2. Ricci WM, Bellabarba C, Lewis R. Angular malalignment after intramedullary nailing of femoral shaft fractures. J Orthop Trauma. 2001; 15(2): 90-95

Question 92

A 70 year old man is undergoing a total knee replacement. The femoral component is ideally placed in 3 degrees of external rotation. Which of the following options is the rationale for this?

To balance the extension gap after the bone cuts

To balance the flexion gap after the bone cuts

To improve patellar tracking

To prevent the joint line being raised

So as not to overstuff the anterior compartment of the knee

ReferencesNormally, the proximal tibia is in slight varus (3 degrees). In TKR, the tibia is cut at 90 degrees (i.e., perpendicular to the mechanical axis). In order to maintain a symmetrical flexion gap, the femoral component must be externally rotated by the same amount to create a symmetrical flexion gap. This allows for balanced ligaments in flexion.

It is a common misconception that the external rotation is to improve patellar tracking, this is simply a beneficial side effect of the cuts and balancing.

Page 63: UKITE 2011

Miller's Review of Orthopaedics (5th edition), pp330-1

Question 93

A 48 year old overweight woman presents with a 2 year history of pain around her ankle. She does recall falling over and injuring her ankle. On clinical exmaniation she is tender behind the medial malleolus and over the sinus tarsi. She is unable to perform a unipedal heel raise. Subtalar joint movements are painful and plain radiographs demonstrate degenerative change in the midtarsal and subtalar joints. Which of the following is the most appropriate management?

Pantalar fusion

Tibilais posterior repair

Tibialis posterior reconstruction

Diagnostic injection

Talonavicular fusionReferences

Core Knowledge in orthopaedics: Foot and ankle. C W DiGiovanni and J Greisberg, Elsevier 2007.Chapter: Adult acquired flatfoot

This lady has history and clinical findings suggesting tibialis posterior insufficiency. However, she has gone on to develop secondary degenerative changes and thus is unlikely to benefit from hindfoot sparing procedures including soft tissue reconstruction and calcaneal osteotomy. Adequate treatment of her symptoms will require selective fusions based on symptomatic relief from diagnostic injection. This may be a triple fusion.

Question 94

A 12-year old boy with no history of trauma presents with lateral knee pain, giving way and a “clunking” sensation on flexion and extension of the knee. What is the most likely diagnosis?

Osteochondritis dissecans

Discoid meniscus

Page 64: UKITE 2011

Anterior Cruciate Ligament rupture

Recurrent patella dislocation

Blount's disease

ReferencesReview of Orthopaedics, Miller 5th edition, chapter 4, section 1/IVNetters Orthopaedics, GreeneCurrent Diagnosis and Treatment in Orthopaedics, SkinnerApleys System of Orthopaedics and Fractures, SolomanOrthoteers WebsiteDiscoid menisci (“popping knee syndrome”)—Can be classified as (1) incomplete, (2) complete, or (3) Wrisberg's variant (Fig. 4–14). Patients may develop mechanical symptoms, or “popping,” with the knee in extension. Plain radiographs may demonstrate a widened joint space, squaring of the lateral femoral condyle, cupping of the lateral tibial plateau, and a hypoplastic lateral intercondylar spine. MRI (three consecutive sagittal images demonstrating a contiguous lateral meniscus) can be helpful and may also demonstrate associated tears. Treatment includes partial meniscectomy (saucerization) for tears, meniscal repair for peripheral detachments (Wrisberg's variant), and simple observation for discoid menisci without tears.

Question 95

Using the Wagner Classification of diabetic foot ulcers, which answer best describes a Grade 3 ulcer?

Gangrene of the entire foot.

A locailised, superficial ulcer.

A deep ulcer to tendon, bone, ligament or joint.

Gangrene of toes or forefoot.

Deep abscess, osteomyelitis.

ReferencesMiller Pg 390, 5th Edition - Disorders of the Foot and Ankle

Question 96

Page 65: UKITE 2011

Which of the factors below does NOT contribute to catastrophic wear in TKR

A flat tibial PE

A kinematic design allowing ample femoral rollback

Congruent PE machining

Irradiation of PE in a vacuum

Polyethylene thickness less than 8mm

ReferencesMiller Table 5-7

Question 97

The nerve of the anterior compartment of the leg is

Common peroneal

Deep peroneal

Saphenous

Superficial peroneal

Sural

ReferencesLast's Anatomy. Regional and Applied. Tenth edition. CS Sinnatamby. The deep peroneal (anterior tibial) nerve. The common peroneal divides into superficial and deep peroneal nerves at the level of the neck of fibula. The deep peroneal enters the anterior compartment of the leg. It is lateral to the anterior tibial vessels in the upper thirds, anterior in the middle thirds and again lateral in the lower third of the leg. The is called 'nervi hesitans' as it begins to cross the anterior tibial vessels from lateral to medial side, hesitates, and comes to lie on their lateral side again.

Page 66: UKITE 2011

Question 98

A 40 year old jumps from the back of a lorry sustaining a fracture to the medial tibial plateau. There is no involvement of the lateral plateau as seen on a CT scan. This can be classified according to Schatzker classification as to what type of injury?

1

2

3

4

5

ReferencesSchatzker J, McBroom R, Bruce D. The tibial plateau fracture. The Toronto experience 1968--1975. Clin Orthop Relat Res. 1979;(138):94-104.

Question 99

The following statements about blood vessels around the hip is correctThe external iliac artery can be injured by anterosuperior quadrant acetabular

screwThe lateral circumflex femoral artery is usually a direct branch of femoral

arteryProfunda femoris is a branch of femoral artery before entering femoral

triangleMedial circumflex femoral artery supplies least blood to the femoral head

Superficial femoral artery continues on lateral side of thigh in Hunter's canal

References

Question 100

The following are symptoms and signs of cauda equina syndrome EXCEPT

Page 67: UKITE 2011

Numbness in the perineum

Severe back pain

An extensor plantar response.

Urinary incontinence

Lax anal sphincter

References2007 Current Medical diagnosis and Treatment, Stephen J. McpheeOxford text book of Surgery, 2nd edition, vol. 3, Peter J. Morris, 2000Short Practice of Surgery, Bailey and Love's, 24th Ed, 2004Cauda equina will exhibits signs of a lower motor neurone lesion

Question 101

Which of the following statement regarding thoracic disc herniations and treatment is true ?Thoracic disc herniation is more common in the upper than lower thoracic

discsDegenerative thoracic discs display less joint instability to that shown in

degenerate lumbar discsThoracic disc replacement surgery although not common is a well

established procedure in the UKAfter thoracic discectomy, fusion is commonly performed to provide

stabilityThoracic disc herniations are usually asymptomatic as the cord to canal ratio

is low

ReferencesNeurosurg Clin N Am. 1993 Jan;4(1):75-90.Thoracic disc herniations.Dietze DD Jr, Fessler RG.The presence of ribs stabilise the thoracic spine even if the disc is removed.

Page 68: UKITE 2011

Therefore disc replacement or fusion is rarely required. Cord compression is well recognised as a complication of thoracic discs due to the high cord to canal ratio.

Question 102

A patient is having a Smith-Petersen osteotomy of the spine for kyphosis. Which of the following is correct?

The procedure shortens the posterior column

A wedge is removed from the posterior half of the vertebral body

An opening wedge osteotomy of the anterior vertebral body is performed

An anterior fusion is never required

This procedure cannot be performed at multiple levels

ReferencesCorrective Osteotomies in Spine SurgeryJ. Brian Gill, MD, MBA; Andrew Levin; Tim Burd; Michael LongleyJ Bone Joint Surg Am. 2008; 90:2509-2520This is a technique for shortening the posterior column thereby correcting forward (positive) saggital plane deformity. You get about 10 degrees per level. The reference above is a good recent review of all the commonly used osteotomy techniques.

Question 103

The circulating blood volume of an 8-year old child is approximately

70ml/Kg

80ml/kg

90ml/kg

100ml/kg

150ml/kg

ReferencesATLS manual, 7th Ed. 2004

Page 69: UKITE 2011

Question 104

In which of the following does endo-chondral ossification occur?

Callus formation during fracture healing

Heterotopic bone formation

Embryonic long bone development

Calcifying cartilage tumors

All of the above

ReferencesBrinker MR. Basic science: Bone. In: Miller MD, ed. Review of Orthopaedics. 3rd ed. Philadelphia, Pa: WB Saunders; 2000:9-12.

Question 105

The following statements regarding traumatic spondylolisthesis of the axis (C2) are true EXCEPT The Levine and Edward's classification system is based only on the lateral

radiograph

Neurological involvement is uncommon

Tends to compress the neural canalType 1 fractures are usually secondary to hyperextension and axial loading

Type II and III fractures are unstable and disrupt the C2/3 motion segment.

ReferencesManagement of traumatic spondylolisthesis of the axis JBJS Am 67A:217-226, 1985* Classificationo Type I+ No angulation+ Up to 3 mm of anterior translation+ Stable

Page 70: UKITE 2011

o Type II+ Angulation > 10 degrees+ Anterior translation > 3 mmo Type III+ Very unstable+ Severe angulation / displacement+ Dislocation of one or both C2-C3 facets* Treatment* Type I : Hard collar for 8 weeks* Type II : SOMI brace after closed for 8 weeks* Type III :o Closed Reduction of dislocated facets may not be possibleo May need ORIF with fusion

Associated with neurology in only 5-10%

Question 106

What is the most common cause for revision of a reversed Total Shoulder Replacement?

Deep infection

Loosening of glenoid component

Loosening of humeral component

Loss of range of motion

Peri-prosthetic fracture

ReferencesBjørg-Tilde S Fevang,Stein A Lie,Leif I Havelin, Arne Skredderstuen, and Ove Furnes. Risk factors for revision after shoulder arthroplasty1,825 shoulder arthroplasties from the Norwegian Arthroplasty Register.Acta Orthop. 2009 February 26; 80(1): 83–91.

Question 107

The safe insertion of acetabular screws in total hip arthroplasty is:

Anteroinferior

Anterosuperior

Page 71: UKITE 2011

Posteroinferior

Posterosuperior

Posterolateral

References

Question 108What is the most likely synovial response to metal on metal volumetric wear debris?

Fibroblast activation

Lymphocyte activation

Macrophage activation

Mast cell activation

Metal allergy

References

Page 72: UKITE 2011

Question 109

The Artery of Adamkiewicz is an important vessel in spinal surgery. A : Anterior (longitudinal) spinal arteryB : Lumbar vesselsC : Right posterior intercostal arteryD : AortaE : Thoracic intercostal vesselsF : Both anterior and posterior intercostal arteriesG : Left posterior intercostal arteryH : Anterior segmental medullary arteryI : Anterior cord syndromeJ : T6-T8K : T9-T10L : L3-L41 : The artery is usually found between which levels?Correct answer: KYour answer: K2 : What is the typical origin vessel of the artery of Adamkiewicz?Correct answer: GYour answer: D

Page 73: UKITE 2011

3 : What is the primary anastomosis of the artery of Adamkiewicz which supplies the spinal cord?Correct answer: AYour answer: F

ReferencesGrays Anatomy - The Anatomical Basis of Clincal Practice. 39th Edition.A of A comes off the left posterior intercostal artery which in turn branches from the aorta.The Adamkiewicz artery is located between T8 and L3, at T9 or T10 in 50%, and coming from the left side in 75% of cases.

Spinal cord ischaemia is rare after segmental vessel ligation. Spinal angiography allows determining the topography of the Adamkiewicz artery safely so the approach can be altered as necessary.

Question 110

Which of the following cytokines and growth factors are important in wound healing?A : Endothelin 1 B : Intracellular signal-regulated kinaseC : Transforming growth factor beta 1D : Matrix metalloproteinase E : Transforming growth factor beta 3 F : Extracellular signal-regulated kinaseG : Connective tissue growth factorH : Transforming growth factor beta 2I : Activin receptor-like kinaseJ : Transforming growth factor beta receptor1 : Which major growth factor is released by damaged platelets in a surgical wound?Correct answer: CYour answer: E2 : Topical application of which growth factor enhances wound healing?Correct answer: CYour answer: H

Page 74: UKITE 2011

3 : Which growth factor can cause fibroblast proliferation in wound healing?Correct answer: GYour answer: G

ReferencesB R Klass, A O Grobbelaar, K J Rolfe. Transforming growth factor ß1 signalling, wound healing and repair: a multifunctional cytokine with clinical implications for wound repair, a delicate balance. Postgrad Med J 2009;85:9-14.

Scarring from surgical incisions, traumatic wounds, burns or infection presents a major clinical problem. It is estimated that 100 million patients a year in the developed world acquire a scar, with a significant proportion of these scars being classified as pathological. The upregulation or abnormal signalling of the multifunctional cytokine transforming growth factor ß1 (TGFß1) has been considered to be the main culprit in the formation of scar tissue because of its ability to induce fibroblasts to synthesise and contract the extracellular matrix (ECM) and induce its downstream mediator, connective tissue growth factor (CTGF; also known as CCN2) and endothelin 1 (ET1).

The reaction to injury is an inherent response resulting in restoration of tissue integrity. Wound healing is often described as a series of events, where soluble mediators from one cell set off a series of events in another cell, resulting in gene transcription and protein translation, culminating in the main phases of wound healing. Briefly, damaged vessels allow the production of a fibrin clot, preventing further blood loss and allowing a provisional matrix for cell migration. Various growth factors and cytokines are released from damaged platelets including TGFß1. TGFß1 is known to cause its own gene transcription, resulting in a positive feedback loop (fig 3), and has been shown to be consistently present throughout the adult wound-healing process. TGFß1 attracts neutrophils, macrophages and fibroblasts into the wound. Re-epithelialisation occurs, with keratinocytes moving across the granulation tissue. Once the keratinocytes have established a barrier, contact inhibition makes them switch phenotype, and they differentiate into stratified squamous keratinising epidermis.The TGFß family of cytokines is an important family of cytokines associated with a number of physiological processes. Aberrant expression of TGFß1 has been shown to be associated with a number of diseases including fibrosis or scarring. This review will discuss how TGFß1 affects gene expression through its intracellular signalling pathways, the role that TGFß1 plays in

Page 75: UKITE 2011

both wound healing and scarring, and the possible clinical implications and potential future treatments to prevent or reduce scarring or fibrosis.

Question 111

With reference to the biomechanics of knee please select the most appropriate answers.A : Anterior Cruciate Ligament (ACL)B : Posterior Cruciate Ligament (PCL)C : Popliteofibular LigamentD : Iliotibial bandE : Popliteal Oblique Ligaments (POL)F : Superficial Medial Collateral Ligament (MCL)G : Deep Medial Collateral Ligament (MCL)H : Lateral Collateral Ligament (LCL)I : MenisciJ : Posterior capsule1 : The primary restraint to internal rotationCorrect answer: AYour answer: A2 : The secondary restraint to posterior translationCorrect answer: HYour answer: H3 : Primary restraint to valgus stress at all angles

Page 76: UKITE 2011

Correct answer: FYour answer: F4 : Provides about 24% of the function as a secondary restraint to anterior translationCorrect answer: DYour answer: D

ReferencesBasic Orthopaedic Sciences The Stanmore Guide. edited by Manoj Ramachandran. 2007

Question 112

Management of Calcaneal Fractures still pose a major dilema for the trauma surgeon. Select the correct answers related to calcaneum fractures.A : 25-40 degreesB : 95-100 degreesC : Essex LoprestiD : HarrisE : Broden's viewF : 40-60 degreesG : Frogleg ViewH : 120-140 degreesI : Neer'sJ : AOK : Sander's1 : What is Bohler's angle in a normal calcaneum?Correct answer: AYour answer: A2 : What radiographic view is used to image the body and the sustenaculum of the Calcaneus?Correct answer: DYour answer:

Page 77: UKITE 2011

D3 : A classification that uses the lateral radiograph to assess articular involvement and joint depression?Correct answer: CYour answer: C

ReferencesRockwood and Green's fractures in adults. 6th Edition.Campbell’s operative Orthopaedics, 11th edition – vol IV, part XIX, chapter 86Studies in fracture patterns, soft-tissue management, and outcomes of calcaneal fractures have given a clear understanding of which injuries are likely to benefit from early surgical intervention and which are likely to have high rates of complications and poor outcomes. Regardless of the treatment, calcaneal fractures are associated with numerous complications and poor outcomes with significant long-term quality-of-life issues.

Calcaneal fractures can be extraarticular (not involving the subtalar joint) or intraarticular (involving the subtalar joint).

Radiographic evaluation of the fracture should include five views. A lateral radiograph is used to assess height loss (loss of Böhler angle) and rotation of the posterior facet. The axial (or Harris) view is made to assess varus position of the tuberosity and width of the heel, which is often a major cause of morbidity for patients particulary with regards to finding comfortable footwear. Anteroposterior and oblique views of the foot are made to assess the anterior process and calcaneocuboid involvement. A single Brodén view, obtained by internally rotating the leg 40 degrees with the ankle in neutral, then angling the beam 10 to 15 degrees cephalad, is made to evaluate congruency of the posterior facet. For surgeons experienced in the care of these fractures, three radiographs may be sufficient, but most often CT scans are obtained to evaluate the injury completely. The scans should be ordered in two planes—the semicoronal plane, oriented perpendicular to the normal position of the posterior facet of the calcaneus, and the axial plane, oriented parallel to the sole of the foot.

Sanders calssification is now commonly used to decide and plan surgical management, but it is based on the CT images only. In particular the location and comminution of posterior facet on coronal CT scan.

Page 78: UKITE 2011

Question 113

Select which tumour most closely matches the description below:A : Osteosarcoma B : EnchondromaC : Ewing's sarcomaD : Periosteal chondromaE : OsteochondromaF : ChondroblastomaG : Aneurysmal bone cystH : Simple bone cystI : Giant cell tumourJ : Chondrosarcoma1 : A malignant tumor of bone in which neoplastic osteoid is produced by a proliferating spindle cell stroma.Correct answer: AYour answer: J2 : A developmental dysplasia of peripheral growth plate which forms a cartilage capped projection of bone found near metaphyses of long bones.Correct answer: EYour answer: E

Page 79: UKITE 2011

3 : A benign and asymptomatic cartilaginous tumor of bone which most often occurs in adolescents or young adults.Correct answer: BYour answer: B4 : This tumour is associated with a translocation between chromosomes 11 and 22Correct answer: CYour answer: C

ReferencesCampbells operative orthopaedics, 11th ed, Vol 1. p 793-810.Osteosarcoma: spindle cell neoplasms that produce osteoid are arbitrarily classified as osteosarcoma. The "classic" osteosarcoma is the most common and usually occurs about the knee in children and young adults. Osteochondroma is a bening surface lesion ariseing from secondary aberrant cartilage(from perichondrial ring) on the surface of bone.Chondromas are benign cartilage lesions, if they are localized to the medularry cavity, they are called enchondromas.

Page 80: UKITE 2011

Question 114

The internervous plane between which pair of nerves is exploited by the following surgical approaches?A : Axillary and SuprascapularB : Musculocutaneous and UlnarC : Median and RadialD : Medial Pectoral and Lateral PectoralE : Axillary and Lateral PectoralF : Posterior Interosseous and RadialG : Ulnar and RadialH : Musculocutaneous and RadialI : Axillary and RadialJ : Median and Ulnar1 : Henry approach to the forearmCorrect answer: CYour answer: B2 : Anterior approach to the shoulderCorrect answer: EYour answer: A3 : Anterior approach to the mid-shaft of humerus

Page 81: UKITE 2011

Correct answer: HYour answer: C

References

Question 115

Choose the appropriate antibiotic to match the given characteristics.A : CefuroximeB : CeftriaxoneC : CephradineD : ClindamycinE : GentamycinF : Amphotericin BG : CiprofloxacinH : Chloramphenicol1 : A third generation cephalosporin.Correct answer: BYour answer: A2 : Achieves high concentration in bone (nearly equals serum concentration after IV administration).Correct answer: DYour answer: D3 : Antibacterial that is unsuitable for use with cement powder as it becomes inactivated during cement polymerisation.Correct answer:

Page 82: UKITE 2011

HYour answer: D4 : Used in the treatment of candida osteomyelitis.Correct answer: FYour answer: F

ReferencesReview of Orthopaedics. Miller. 4th edition.

Question 116

Choose the best surgical treatment option for the given fractureA : Cannulated Hip screwsB : Dynamic hip screwC : Total Hip ReplacementD : Cemented HemiarthroplastyE : Intramedullary Hip screwF : Non operative managementG : Internal FixationH : Bipolar HemiarthroplastyI : Uncemented Hemiarthroplasty1 : An 85 year old lady, poorly mobile with a displaced intracapsular fracture neck of femur.Correct answer: DYour answer: H2 : A 75 year old active gentleman with a three-part pertrochanteric fracture neck of femur.Correct answer: BYour answer: E

Page 83: UKITE 2011

3 : A 60 year old gentleman with Rheumatoid arthritis and a displaced intracapsular fracture neck of femur.Correct answer: CYour answer: C

ReferencesNICE guidelines on treatment oh Hip fractures

Question 117

Which is most likely vessel to be injured during the following procedures?A : Perforating branches of profunda femoris arteryB : Superficial Femoral arteryC : Profunda femoris arteryD : Transverse branch of lateral circumflex arteryE : Ascending branch of the medial circumflex arteryF : Superior gluteal artery G : Inferior gluteal arteryH : Superficial circumflex iliac arteryI : Pudendal arteryJ : External iliac artery1 : Lateral approach to the femur for a dynamic hip screw following a closed reduction of an intertrochanteric fracture.Correct answer: AYour answer: G2 : Insertion of a straight femoral intramedullary nail for a subtrochanteric fracture.Correct answer: EYour answer:

Page 84: UKITE 2011

E3 : Initial approach for a trochanteric entry cephalo-medullary nailCorrect answer: FYour answer: F

ReferencesCambell's operative orthopaedics, Hoppenfeld's Surgical exposures in orthopaedics, Moein et al Injury 2005, Eksioglu et al CORR 2003, Ozsoy JBJS-Am 2007

Question 118

At which level in the physis do the following pathological processes occurA : Primary spongiosaB : Secondary spongiosaC : Zone of provisional calcificationD : Proliferative zoneE : Reserve zoneF : Degenerative zoneG : Maturation zoneH : Secondary bony epiphysis1 : RicketsCorrect answer: CYour answer: C2 : Acute haematogenous osteomyelitisCorrect answer: AYour answer: A3 : AchondroplasiaCorrect answer: D

Page 85: UKITE 2011

Your answer: D4 : Osteogenesis imperfectaCorrect answer: BYour answer: F

ReferencesReview of Othopaedics 5th edition - MD MillerFigure 1-13 on page 12 lists zones, histology, function, exemplary diseases and defects.

Question 119

In digital flexor tendon sheath InfectionA : Eikenella corrodensB : Index, middle and ring fingersC : Pasteurella multocidaD : Parona’s signE : Thumb, index and little fingersF : Dickson-Wright’s signG : Kanavel’s signH : Thumb, ring and little fingersI : Staphylococcus aureusJ : Felon’s sign1 : The digits most commonly involved areCorrect answer: BYour answer: B2 : The most common causative agent following a human bite isCorrect answer: IYour answer: I

Page 86: UKITE 2011

3 : The name given to the four cardinal signs (flexed position of finger, symetrical enlargement of the whole finger, excessive tenderness over the course of the sheath and excruciating pain with passive finger extension) isCorrect answer: GYour answer: G

References1) Index, Middle and Ring fimnger flexor sheaths are most commonly

involved.2) The most common bacterial agent is s.aureus. A high index of suspicion of Pasteurella multocida should be made if the infection develops within 24hrs of a cat bite. Eikenella corrodens is seen with higher incidence following human bite wounds, but s.aureus is still the most common organism. 3) Described by Allen B Kanavel in 1912. Parona’s space is a sub-tendinous space of the wrist – Pus in FPL sheath can ascend in the radial bursa and rupture into this space. A felon is a closed space infection of the finger tip pulp. Dickson-Wright was one of the first to suggest post-operative sheath irrigation back in 1943.

Page 87: UKITE 2011

Question 120

Tumours around the knee in the young adult. Select the most appropriate first line treatment modality of each of the following conditionsA : Above knee amputationB : Intra-articular chemotherapyC : Marginal resectionD : Neoadjuvent chemotherapyE : Radical resectionF : RadiotherapyG : Wide local excision with endoprosthetic replacementH : Wide local excision with joint preservationI : Intra-lesional procedure1 : Enneking IIa Osteosarcoma of distal femoral metaphysisCorrect answer: DYour answer: D2 : Localised Nodular PVNSCorrect answer: CYour answer: E3 : Large Distal femoral Aneurysmal Bone Cyst

Page 88: UKITE 2011

Correct answer: IYour answer: I

ReferencesReview of orthopaedics, fifth edition. Mark D Miller. Saunders.Osteosarcomas are primarily treated with Neoadjuvent chemotherapy as this is the most important treatment modality for improving survival. This is thought to reduce the presence of early pulmonary micrometastasis.

Nodular PVNS can be resected locally unless it has breached the joint capsule. There is however a high recurrence rate and re-resections are often combined with adjuvent radiotherapy.

Aneurysmal bone cysts and Giant Cell Tumours can be treated with curettage and impaction bone grafting - a form of intralesional resection.

Question 121

Regarding the biomechanical behaviour of the following - match the appropriate answers to the statementsA : 10 foldB : 8 foldC : 16 foldD : 4 foldE : 2 foldF : 0.5 foldG : 20 foldH : 1 foldI : 5 foldJ : 3 fold1 : For a given material, doubling the plate thickness increases its rigidityCorrect answer: BYour answer: B2 : For a given material, doubling the radius of a solid nail increases its rigidityCorrect answer: CYour answer: C

Page 89: UKITE 2011

3 : For a given thickness of plate, how many times is a titanium plate rigid in comparison to a stainless steel plate (approximately)Correct answer: FYour answer: F

References

Basic Orthopaedic Sciences - The Stanmore GuideDoubling the thickness of a plate (rectangular construct) results in an increased SMA and consequently increased rigidity by the third power of the multiplying factor which in this case is 2 raised to the power of 3 = 8.Doubling the thickness of a rod results in an increased PMA and consequently increased rigidity by the fourth power i.e. 2 raised to the power of 4 = 16.

Rigidity is defined as SMA multiplied by the Youngs modulus. Youngs modulus of steel is roughly half that of stainless steel. Hence for a given plate thickness, titanium plates are half as rigid as stainless steel.

Page 90: UKITE 2011

Question 122

The following is true concerning each of the below listed approaches to the hip joint:A : Is an extensile approachB : Utilises the interval between tensor fascia lata and gluteus mediusC : Deep dissection is between adductor magnus and adductor brevisD : Uses the interval between sartorius and the femoral sheathE : Would require repair of the inguinal ligamentF : Splits muscle innervated by the inferior gluteal nerveG : Is recognised to put the superior gluteal nerve at increased risk of injuryH : Is recognised to put the lateral circumflex femoral vessel at an increased risk of injuryI : Allows the possibility of increasing muscle tension without altering centre of rotation, neck length or offset1 : The transtrochanteric approachCorrect answer: IYour answer: I2 : The anterior approach (Smith Peterson)Correct answer: AYour answer: B

Page 91: UKITE 2011

3 : The medial approach (Ludloff)Correct answer: CYour answer: C4 : The posterior approachCorrect answer: FYour answer: A

ReferencesSurgical exposure in Orthopaedics. The Anatomic Approach, Stanley Hoppenfeld and Piet deBoer. Third Edition. Lippincott Williams and Wilkins. page 365-452

Sir John Charnley pioneered the use of the transtrochanteric in the 1950s. This approach allows an extensile exposure, advancement of the trochanter, and proper soft tissue tensioning. It aids the correct orientation of prosthetic implants and allows the possibility of increasing muscle tension without altering centre of rotation, neck length or offset.

The Smith Paterson appraoch is an extensile approach and exploits the IN plane between the sartorius and tensor fascia lata to penetrate the outer layer of the joint musculature.

The medial approach (Ludloff) utilizes the deep plane of dissection between the adductor brevis and adductor magnus. The superficial dissection does not exploit and IN plane, since both adductor longus and magnus are innervated by the anterior division of the obturator nerve.

Page 92: UKITE 2011

Question 123

With regards to finger replantationA : Almost any part in a child, multiple digits, amputation of thumb B : Almost any part in a child, amputation at multiple levels of digits, amputation proximal to FDSC : Almost any part in a child, amputation distal to DIPJ, crush or mangled partsD : Bone, extensor tendon, flexor tendon, arteries, nerves, veins and skinE : Bone, veins, nerves, skin, flexor tendon, arteries and extensor tendonF : Bone, skin, vein, artery, extensor tendon, flexor tendon, nervesG : Thumb, long, ring, small and index fingerH : Index, small, ring, long and thumbI : Index, long, ring, small and thumb1 : The favourable indications for replantation of finger areCorrect answer: AYour answer: A2 : The ideal operative sequence of replantation in achieving a higher viability rate would beCorrect answer: DYour answer: D

Page 93: UKITE 2011

3 : The priority for digit replantation in multiple digit amputation must be in the order ofCorrect answer: GYour answer: G

ReferencesTamai, S: Twents years experience of limb replantation- review of 293 upper limb replants. J Hand Surgery(Am)7:549-555, 1982

Urbaniak JR, Roth JH< Nunley JA et alThe results of replantation after amputation of a single finger. JBJS (Am)67:611-619, 1985

1)The favourable indicatons for replantation of finger are any part in a child, thumb and multiple digits and NOT multiple level

2)The ideal sequence of replantation is Bone first then extensor tendon followed by flxor tendon, arteries , nerves ,veins and finally skin

Question 124

With regards to bony development, which answer is most appropriate for the following statements?A : Triradate cartilageB : Superior femoral physisC : Greater trochanteric physisD : Lesser trochanteric physisE : Ischial physisF : Femoral head ossification centreG : Greater trochanter ossification centreH : Lesser trochanter ossification centreI : Ischial tuberosity ossification centreJ : Pubis ossification centre1 : Which structure appears around the age of 6-12 months?Correct answer: FYour answer: F2 : Which structure appears around puberty?Correct answer: I

Page 94: UKITE 2011

Your answer: D3 : Which structure appears around 5 years of age?Correct answer: GYour answer: J4 : Which structure fuses between 20 and 22 years of age?Correct answer: EYour answer: J

ReferencesGray’s anatomy

Question 125

With regards to the cruciate and collateral ligaments of the knee, which structure needs to be damaged to result inA : Superficial MCLB : Anterior Cruciate LigamentC : Posterior Cruciate LigamentD : Deep Part of MCLE : Lateral Collateral ligamentF : Popliteus TendonG : Medial MeniscusH : Lateral meniscusI : Lateral Meniscofemoral LigamentJ : Gastronemius Tendon1 : Loss of roll-glide mechanism Correct answer: BYour answer: A2 : Positive Valgus stress testCorrect answer: AYour answer: A3 : Loss of screw home mechanism and increased tibial external RotationCorrect answer:

Page 95: UKITE 2011

FYour answer: F

ReferencesGoldblatt JP, Richmond JC. Anatomy and biomechanics of the knee. Operative techniques in Sports Medicine,2003; 11(3): 172-186. Martelli S, Pinskerova V. The shapes of the tibial and femoral articular surfaces in relation to tibiofemoral movement. Freeman MAR, Pinskerova V. The movement of the normal tibio-femoral joint. J Biomech,2005; 38:197-208. Superficial part of MCL is the more important contributor to stability. Cutting the superficial part results in joint widening.Apart from anterior translation of tibia, ACL deficiency also results in loss of roll-glide mechanism at the joint. Rolling predominates in initial flexion, followed by sudden posterior shift of contact point (this is the basis for the Pivot shift test). Popliteus has important role in screw home mechanism. Sectioning of Popliteus results in marked increase in tibial ER in 90 degrees flexion.

Question 126

For the following clinical scenarios select the most appropriate spinal procedureA : VertebroplastyB : CorpectomyC : Instrumented fusionD : Spinal stabilisationE : Growth RodsF : Interbody fusionG : Harrington rodsH : 360 degree fusionI : Pedicle subtraction osteotomy (PSO)J : Brace application and non operative management1 : Painful osteoporotic vertebral compression fracture. STIR images on the MRI scan show a cleft with fluid at the site of the fracture.Correct answer: A2 : May be used to correct a significant kyphotic deformityCorrect answer: I

Page 96: UKITE 2011

Your answer: I3 : A procedure that involves removal of a vertebral body often with subsequent cage reconstruction.Correct answer: BYour answer: B4 : Treatment for a flexible adolescent idiopathic scoliosis curve with a Cobb angle of 40 degrees in a premenarchal girl.Correct answer: CYour answer: C

References

Kallmes DF, Comstock BA, Heagerty PJ, et al. (August 2009). "A randomized trial of vertebroplasty for osteoporotic spinal fractures". N. Engl. J. Med. 361 (6): 569–79

Curve Progession in Idiopathic ScoliosisWeinstein et alJBJS(Am) 1983 Vol 65(4) 447-455

1) Vertebroplasty or kyphoplasty may be used to treat osteoporotic fractures. If you can see fluid in a cleft at the fracture site on a STIR sequence then it suggests that treatment will be successful.

2) PSO or mutliple Ponte osteotomies are used to correct saggital balance which are significant eg 30 degrees

3) Curves greater than 40 degrees in a premenarchal girl are likely to progress

Question 127

When designing a study which factor needs to be considered ifA : Power analysisB : Null hypothesis

Page 97: UKITE 2011

C : Parametric testD : Type II errorE : Type I errorF : Linear regressionG : Independent variablesH : MatchingI : Randomisation1 : You are trying to calculate the sample size neededCorrect answer: AYour answer: A2 : You are estimating the rate of false positive resultsCorrect answer: EYour answer: E3 : You are comparing two continous variablesCorrect answer: FYour answer: F

ReferencesA power analysis is a method of determining the number of patients required in a study to have a reasonable chance of showing a difference if one exists.

The null hypothesis is an assumption that any difference seen is purely by chance. Studies are designed to either prove or disprove this assumption.

A parametric test assumes data is sampled from a particular form of distribution such as a normal distribution. Non-parametric tests make no such assumption

Errors arise when accepting or rejecting the null hypothesis. A type I (alpha) error occurs when a difference is found but in reality there is not a difference. A type II (beta) error occurs when no difference is found but a difference does exist.

Linear regression Correlation is a term used to describe the relationship between two parameters. Linear regression is when the relationship can be plotted on a straight line such as with parametric data. Regression can also be curved or logistic.

Page 98: UKITE 2011

For variables to be independent, there needs to be no chance that a subject could appear in both groups being compared. An unpaired T test would be used to compare independent variables provided they follow a normal distribution.

Matching is a process of identifying subjects in different groups that have certain similar characteristics (eg. Age, sex, co-morbidities)

Randomisation ensures that all prognostic variables, known and unknown, will be distributed evenly among the treatment groups. Randomisation can be simple (eg. Computer-generated tables), stratified, or block.

Basic Orthopaedic Sciences. The Stanmore Guide. Ramachandran M. Hodder Arnold.

Question 128

The following pathologies are associated with which of these clinical conditions?A : Cerebral PalsyB : Rett's syndromeC : PoliomyelitisD : Guillain-Barre syndromeE : Charcot-Marie-Tooth DiseaseF : Freidrich's AtaxiaG : Spinal Muscular AtrophyH : Duchenne's DystrophyI : Becker's DystrophyJ : Werdnig-Hoffmann Disease1 : Genetically determined demyelination and/or axonal degeneration in peripheral nervesCorrect answer: EYour answer: E2 : Periventricular LeukomalaciaCorrect answer: AYour answer: I

Page 99: UKITE 2011

3 : Autoimmune mediated demyelination and/or axonal destruction in peripheral nervesCorrect answer: DYour answer: D

ReferencesTachdjian's Pediatric Orthopaedics. 4th Edition, 2008. Vol 2, Neuromuscular Disorders: pp 1675-1674.

CMT has been classically divided into demyelinating and axonal forms. But research indicates that demyelination renders the axon susceptible to degeneration and henec the 2 pictures can co-exist.

Periventricular leukomalacia and intra and periventricular haemorrhages are frequent MRI finidings in Cerebral palsy. The former results from an ischemic insult to the arterial watershed area close to the ventricular walls.

GB Syndrome is now the commonest cause of acute flaccid paralysis in children in the west. It is characterised by symmetric motor and sensory paresis of the limbs and at times the trunk. The disease is auto-immune and directed against peripheral nervous system myelin, axon or both. It is triggered by a preceding bacterial or viral infection.

Question 129

Concerning lumbar intervertebral discsA : Type III CollagenB : Type VI CollagenC : Type II CollagenD : Type IV CollagenE : Type X CollagenF : Type I CollagenG : Type XI CollagenH : Type IX Collagen1 : Which type of collagen is most prevalent in the nucleus pulposus affected by age related change?Correct answer: FYour answer:

Page 100: UKITE 2011

A2 : Which type of collagen is most prevalent in the nucleus pulposus of the normal disc?Correct answer: CYour answer: C3 : Which type of collagen is involved in the cross linking of aggregates in the intervertebral disc?Correct answer: HYour answer: H

References1. Hadjipavlou AG, Tzermiadianos MN, Bogduk N, Zindrick MR. The pathophysiology of disc degeneration: a critical review. J Bone Joint Surg Br 2008;90-10:1261-70.

1. The collagen content of the nucleus increases and changes from type II to type I Collagen rendering the nucleus more fibrous during the ageing process[1]. The concentration of cells and proteoglycans however decrease within the disc with age.

2. Type IX collagen cross links aggregates which are held together by type II collagen.

Page 101: UKITE 2011

Question 130

With regards to bone tumours choose the most appropriate response for each of the followingA : OsteosarcomaB : ChondrosarcomaC : EnchondromaD : Ewings sarcomaE : Osteoid osteomaF : Unicameral bone cystG : Non-ossifying fibromaH : Giant cell tumourI : ChondroblastomaJ : Metastatic bone tumour1 : Associated with Paget diseaseCorrect answer: AYour answer: A2 : Lytic lesion in adults that can extend to subchondral area, narrow zone of transitionCorrect answer: HYour answer: H

Page 102: UKITE 2011

3 : Mirel's scoreCorrect answer: JYour answer: J4 : Can be treated by radio-frequency ablationCorrect answer: EYour answer: E

ReferencesApley'sMillerRamachandran Basic Sciencs

Question 131

Match the following biomechanical terms for stress-strain behaviour of ligaments to their descriptionA : Plastic deformationB : Toe-inC : FractureD : DipsE : Elastic deformationF : CreepG : HysteresisH : Stress relaxationI : Compressive failureJ : Ligament remodelling1 : The first region in the stress-strain curve of ligament which shows a non-linear relationshipCorrect answer: BYour answer: B2 : This behaviour is applied in the application of plaster casts for deformity correctionCorrect answer: FYour answer: F

Page 103: UKITE 2011

3 : Small force disruptions sometimes observed at the end of the linear region of the stress-strain curveCorrect answer: DYour answer: H

ReferencesBasic Orthopaedic Sciences - The Stanmore Guide

The toe-in region is due to the straightening of the crimped fibres. The dips that are seen towards the end of the linear portion of the curve are caused by early sequential failure of a few greatly stretched fibre bundles.

The three main features of the visco-elastic material are stress relaxation, creep and hysteresis.Stress relaxation is described as decrease in stress when subjected to a constant strain over time.Creep is described as increase in deformation (strain) when a constant load is applied over time.Hysteresis is described net internal energy loss during loading and unloading of the stress-strain curve.

Question 132

With regard to nerves at risk during surgical approaches to the hipA : Pudendal nerveB : Nerve to obturator internus C : Superior gluteal nerveD : Inferior gluteal nerveE : Sciatic nerveF : Posterior femoral cutaneous nerveG : Lateral femoral cutaneous nerveH : Nerve to quadratus femorisI : The cluneal nervesJ : The anterior and posterior divisions of the obturator nerve1 : Which nerve may be injured during the medial approach?Correct answer: JYour answer: J2 : Which nerve may be injured during the anterior approach?

Page 104: UKITE 2011

Correct answer: GYour answer: G3 : Which nerve may be injured during the lateral approach?Correct answer: CYour answer: C

ReferencesHoppenfeld S, deBoer P. Surgical exposures in orthopaedics the anatomic approach. 3rd ed. Lippincott Williams and Wilkins 2003

The anatomy of the medial approach is the anatomy of the adductor compartment of the thigh. The obturator nerve is derived from the anterior division of the L2-L4 nerve, and divides in the obturator notch into anterior and posterior divisions. The nerve lies on the anterior surface of the adductor brevis. The posterior division of the obturator nerve runs dially on the surface of the adductor magnus.

The anterior approach internervous plane lies between the sartorius and the tensor fascia lata. The lateral femoral cutaneous nerve passes either over, behind or through the sartorius muscle.

The lateral approach allows exposure of the hip joint for joint replacement. The superior gluteal nerve runs between the gluteas medius and minimus 3-5cm above the greater trochanter and can be damaged with proximal dissection.

Page 105: UKITE 2011

Question 133

Which of the following options best describes these fracture configurations?A : Myerson's type AB : Essex Lopresti lesionC : Sanders type 3.D : Supination adduction injury.E : Myerson's type B.F : Gartland type II.G : Supination-External rotation injury.H : Milch type I.I : Kilfoyle type III.J : Milch type II.1 : Transverse avulsion fracture of the lateral malleolus below the syndesmosis with a vertical fracture of the medial malleolus.Correct answer: DYour answer: D2 : Lateral or dorsoplantar displacement of all five metatarsals with or without fracture of the base of second metatarsal.Correct answer: AYour answer: B

Page 106: UKITE 2011

3 : The fracture line traverses the capitellar ossification centre, into the capitellar trochlear groove.Correct answer: HYour answer: H

ReferencesCampbell's Operative Orthopaedics 10th EditionLauge-Hansen, N. :Ankelbrud. I. Genetisk diagnose og reposition. Dissertation, Copenhagen, Munksgaard, 1942Lauge-Hansen, N. “Ligamentous” ankle fractures. Diagnosis and treatment. Acta Chir. Scand. 97:544, 1949Myerson MS, Fisher RT, Burgess AR, Kenzora JE. Fracture dislocations of the tarsometatarsal joints: end results correlated with pathology and treatment. Foot & Ankle. 1986 Apr; 6(5):225-42Fractures of the medial epicondyle and epicondyle of the elbow in children. Kilfoyle R. CORR 1965 July; 41: 43-50Fractures and fracture dislocations of the humeral condyles. Milch H. 1964 Sept; 4(5):592-607.Inta-articular fractures of the calcaneus: Present state of the art. Sanders R. JOT 1992 June; 6(2): 252-265.Management of supracondylar fractures of the humerus in children. Gartland JJ. Surg Gynecol. Obstet. 1959 Aug; 109(2): 145-54.Fractures of the radial head with distal radioulnar dislocation. Essex-Lopresti P. JBJS (Br) 1951; 33: 244-250.

The Lauge Hansen classification of ankle fractures consists of 4 different types based on the position of the foot and the direction of dislocating force momentum causing the fracture. Supination-External rotation is the commonest type of injury seen with an oblique fracture of the distal fibula at the level of the syndesmosis with or without a fracture of the medial malleolus. Pronation-Abduction, Pronation-External rotation and Supination-Adduction are the remaining three types, the latter consisting of a transverse fracture of the distal fibula inferior to the syndesmosis combined with a vertical shear fracture of the medial maleolus.Quenu and Kuss (1909) described homolateral, isolated and divergent patterns of Lisfranc injury. This was modified in 1982 by Hardcastle to Total incongruity, partial incongruity and divergent. Myerson (1986) further modified this classification into type A (total incongruity- either lateral or dorsoplantar), Type B1 (Partial incongruity- medial dislocation) and B2 (Partial incongruity- lateral dislocation) and C1(divergent- partial displacement) and C2 (divergent- total displacement)

Page 107: UKITE 2011

Milch described lateral condyle fractures of the distal humerus in children.In type 1 fractures the fracture line courses medial traversing the capitellar ossification centre into the capitellar trochlear groove. This is the rarer, more stable configuration. Type 2 fractures are more common and the fracture line extends into the area of the trochlea and produces inherent instability of the elbow.Sanders classification describes calcaneal fractures, Gartland classified supracondylar fractures and Kilfoyle described medial condyle fractures in children.The Essex Lopresti lesion is a radial head fracture in association with a dislocation of the distal radio-ulnar joint and interosseus membrane disruption.

Question 134

Regarding surgical approach for anterior release and posterior correction in scoliosisA : Anterior spinal arteryB : Vertebral arteryC : Segmental vesselsD : Long Thoracic NerveE : Latissimus dorsiF : Serratus anteriorG : Serratus posteriorH : MultifidusI : ConvexityJ : Concavity1 : What structure is at risk during deep exposure of vertebrae for anterior release?Correct answer: CYour answer: A2 : One of the muscles in the of the deep layer of backCorrect answer: HYour answer: F

Page 108: UKITE 2011

3 : Should the anterior approach to a scoliosis be onto the concavity or convexity of the curve?Correct answer: IYour answer: I

ReferencesHoppenfeld and De Boer - Surgical Approaches in OrthopaedicsSegmental vessels lie over the vertebral bodies and are at risk during dissection through overlying fascia whilst gaining access to the discs.Multifidus is a deep back muscle although, in vivo, it is often difficult to distinguish clearly.Anterior approach should be to the convexity of a curve in order to allow maximum access to disc spaces.

Question 135

With regards to testing of neuromuscular conditions in children,A : MRI scan B : Alpha-fetoprotein C : Alanine TransaminaseD : TyrosinaseE : Computerized gait analysisF : EMGG : Creatine kinaseH : AldolaseI : TroponinsJ : Somato-sensory Evoked Potentials (SSEP)1 : This helps in the diagnosis of Cerebral Palsy.Correct answer: AYour answer: A2 : Its serum level can be 20 to 200 times above normal in Duchenne's muscular dystrophy.Correct answer: GYour answer: D3 : This is another enzyme raised in muscular dystrophy.

Page 109: UKITE 2011

Correct answer: HYour answer: H

ReferencesTachdjian's Pediatric Orthopaedics. 4th Edition, 2008. Disorders of the Brain: pp 1275 -1397.

Tachdjian's Pediatric Orthopaedics. 4th Edition, 2008. Muscle Diseases: pp 1621 -1674.

Question 136

For each of the clinical scenarios choose the most likely pathogen A : Staphylococcus AureusB : Staphylococcus EpidermidisC : Streptococcus PyogenaseD : Streptococcus PneumoniaeE : Nieseria GonorrhoeaF : Mycobacterium TuberculosisG : Treponema PallidumH : Pseudomonas AeruginosaI : Salmonella TyphiJ : Escherichia Coli1 : A 25 year-old man presents with thoracolumbar pain. An MRI suggests discitis at L1.Correct answer: AYour answer: A2 : A 45 year-old lady presents with a discharging wound 2 weeks following a posterior lumbar inter-body fusion. Urine dipstick suggests a UTICorrect answer: JYour answer: J

Page 110: UKITE 2011

3 : A 40 year-old lady with 3 months of thoracic back pain. Examination reveals generalised lymphadenopathy and a lump on the left 9th rib at the anterior axillary line. Urine dip showed plenty of white cells but no growth on cultures.Correct answer: FYour answer: F

Referenceshttp://emedicine.medscape.com/article/1263845-overview - (good overview)

Spinal Infections - Govender JBJS(Br) 2005 Vol 87-B, Issue 11, 1454-1458

1. Staphylococcus Aureus is the commonest pathogen involved in discitis

2. Escherichia coli and Proteus species are more common in patients with UTIs.

3. The chronicity, collar stud abscess (lump on the rib) and the sterile pyuria suggest TB

n.b. In IV drug abusers, Klebsiella and Pseudomonas are commonly seen but not as frequently as S. Aureus which is still the commonest cause of discitis.

Page 111: UKITE 2011

Question 137

With regards arthroscopy of the shoulder, choose the most appropriate responseA : Cephalic veinB : Subclavian arteryC : Posterior circumflex arteryD : Long thoracic nerveE : Thoracoacromial arteryF : Axillary nerveG : Suprascapular nerveH : Supraclavicular nerveI : Musculocutaneous nerveJ : Supraclavicular artery1 : What structure can be damaged when the trocar is pushed medially on the neck of scapula during arthroscopic Bankart capsulorraphy?Correct answer: GYour answer: G2 : What nerve is at risk when using a lateral portal for subacromial decompression?Correct answer: FYour answer:

Page 112: UKITE 2011

F3 : Brisk bleeding during arthroscopic subacromial decompression is most likely from which vessel?Correct answer: EYour answer: E4 : Anterior portal placement can lead to injury to this vascular structureCorrect answer: AYour answer: J

References

Campbells operative orthopaedics, 11 th edition, mosby elservier pages 2923-2993

Anatomic risks of shoulder arthroscopy portals: anatomic cadaveric study of 12 portals.Meyer M, Graveleau N, Hardy P, Landreau P. 2007 May;23(5):529-36.

Page 113: UKITE 2011

Question 138

With regards to Vitamin D synthesis A : PTHB : Calcitonin C : KidneyD : Skin E : BoneF : LiverG : Osteoprotegerin H : 25-hydroxyvitamin D3 1-alpha-hydroxylaseI : Vitamin D-25 hydroxylase J : P450 mixed function oxidase 1 : Hydroxylation of cholecalciferol occurs in this organCorrect answer: FYour answer: F2 : Hormone that regulates 1,25 – dihydroxyvitamin D synthesis.Correct answer: AYour answer: B3 : Enzyme that converts vitamin D into calcidiol.Correct answer: IYour answer:

Page 114: UKITE 2011

I

References

AAOS Orthopaedic Basic ScienceIn the skin, 7-dehydrocholesterol, a derivative of cholesterol, is photolyzed by ultraviolet light into previtamin D3. Previtamin D3 spontaneously isomerizes to vitamin D3 (cholecalciferol). Whether it is made in the skin or ingested, cholecalciferol is hydroxylated in the liver to form 25-hydroxycholecalciferol (calcidiol or 25(OH)D). This reaction is catalyzed by the microsomal enzyme vitamin D 25-hydroxylase, which is produced by hepatocytes. Once made, the product is released into the plasma, where it is bound to an a-globulin, vitamin D binding protein. Calcidiol is transported to the proximal tubules of the kidneys, where it is hydroxylated to form calcitriol (aka 1,25-dihydroxycholecalciferol and abbreviated to 1,25(OH)2D). This product is a potent ligand of the vitamin D receptor (VDR), which mediates most of the physiological actions of the vitamin. The conversion of calcidiol to calcitriol is catalyzed by the enzyme 25-hydroxyvitamin D3 1-alpha-hydroxylase, the levels of which are increased by parathyroid hormone (and additionally by low calcium or phosphate).

Page 115: UKITE 2011

Question 139

With reference to calcium metabolismA : HypocalcaemiaB : T-score is >2.5 SD below the mean C : Raised alkaline phosphatase, raised parathyroid hormone and low urinary calciumD : HypercalcaemiaE : T-score is <2.5 SD below the meanF : Z-score is >2.5 SD below the meanG : Raised alkaline phosphatase, low parathyroid hormone and low urinary calciumH : Raised alkaline phosphatase, raised parathyroid hormone and high urinary calciumI : Z-score is between 1 and 2.5 SD below the meanJ : Increased phosphate levels1 : Is a cause of muscular tetanyCorrect answer: AYour answer: A2 : Osteoporosis can be defined byCorrect answer: BYour answer: E3 : Nutritional rickets can be identified by the following blood tests

Page 116: UKITE 2011

Correct answer: CYour answer: C

ReferencesWHO classification of normal bone mineral density is within 1 standard deviation of the mean. Osteopenia is considered when the BMD is 1-2.5 SD below the mean and osteoporosis is considered when BMD is > 2.5 SD below the mean.Urinary calcium is used to differentiate between primary hyperparathyroidism and nutritional rickets. In nutritional rickets, urinary calcium is low while in the other it is high.